Download A 36-year-old woman with a history of migraine headaches comes to

Survey
yes no Was this document useful for you?
   Thank you for your participation!

* Your assessment is very important for improving the workof artificial intelligence, which forms the content of this project

Document related concepts

Dental emergency wikipedia , lookup

Prenatal testing wikipedia , lookup

Medical ethics wikipedia , lookup

Patient safety wikipedia , lookup

Adherence (medicine) wikipedia , lookup

Lumbar puncture wikipedia , lookup

Transcript
A 36-year-old woman with a history of migraine headaches comes to the clinic
complaining of severe left-sided headaches that are often associated with
nausea and sensitivity to light. When they occur they are terribly painful and last
about 12-18 hours. She has 6-8 attacks per month. She also tells you that
occassionally, when she gets headaches, she has trouble with the vision in the
eye on the side of the headache. She currently has no headache and no visual
changes. Physical examination is unremarkable. The most appropriate
intervention at this time is to
A. do nothing at this time
B. increase the dose of her migraine therapy regimen
C. order an erythrocyte sedimentation rate
D. refer the patient for a CT scan of her head
E. refer the patient for a temporal artery biopsy
Explanation:
The correct answer is C. The diagnosis of headache is often a challenge. This
patient has many classical signs and symptoms of migraine. However, her
associated visual loss, which is not described as an "aura" is troubling and
should trigger concern for a possibly different and more severe diagnosis. This
pattern is suggestive, if not worrisome, for temporal arteritis. Since she is having
no current symptoms, the first step in evaluating for this condition is to order an
ESR. Prednisone should be considered to prevent blindness if you strongly
believe that this diagnosis is possible.
Believing that no action is indicated at this time (choice A) is a common mistake
that is made by young physicians because the tendency is to think that patients
such as this are somewhat hypochondriacal.
Having the patient increase the dose of her migraine therapy regimen (choice B)
is not appropriate because she is having new symptoms that are not clearly
related to a migraine. Specifically, visual "aura" is common with migraine, but
loss of vision is not and therefore prompts additional investigation.
Referring the patient for a CT scan of her head (choice D) is not indicated, as
there is no evidence from this patients' history that warrants an evaluation for
any intracranial process.
Referring the patient for a temporal artery biopsy (choice E) would be more
sensitive and specific for the diagnosis of TA, but as a screening tool for disease
it is too invasive.
A 20-year-old woman comes to the office complaining of a 3-year history of headaches. She
describes them as a frontal, throbbing pain, which may be on either side of the head and is
worse when she wakes up in the morning, though it does not wake her from sleep. She
decided to come to the office today because the frequency of the headaches had been
increasing over the past 6 months and for the last month she has had a headache every
single day. She usually takes 650 mg of acetaminophen and 200 mg of ibuprofen every 4
hours during the day. These do provide some relief. She has no other symptoms. Physical
1
and neurologic examination is completely normal. The next most appropriate step in
managing this case is to
A. perform a lumbar puncture
B. prescribe amitriptyline
C. prescribe sumatriptan
D. slowly taper down both the acetaminophen and ibuprofen, prescribing a small
supply of acetaminophen/butalbital/caffeine for emergencies and see her back in 2
weeks
E. stop the acetaminophen and increase the dose of ibuprofen
Explanation:
The correct answer is D. This is a typical presentation for analgesic withdrawal headache.
She must be weaned off the daily use of analgesics for her headaches. There may be an
underlying condition such as migraine or tension type headache, but the analgesics must
first be stopped to determine this.
If a headache of intracranial hypertension were suspected a lumbar puncture (choice A)
might be indicated, but this case seems to be analgesic withdrawal headaches.
Both analgesics must be stopped so stopping the acetaminophen and increasing the dose
of ibuprofen (choice E) is incorrect.
Amitriptyline (choice B) may be used for migraine prophylaxis, but she must be off the daily
analgesics before we make that diagnosis.
Sumatriptan (choice C) may be used for acute treatment of migraine, but she must be off
the daily analgesics before we make that diagnosis.
As the longtime primary care physician of an 85-year-old woman, you are asked
to help mediate care between the family and the hospitalist. The patient
sustained an anoxic brain injury during an in-hospital cardiac arrest one week
ago where the patient had a pulseless period for at least 5 minutes. On physical
exam, the vital signs are normal and stable with the assistance of continuous
mechanical ventilation. Pupillary and corneal reflexes are present bilaterally.
There is episodic decorticate rigidity, but no purposeful movement present. An
electroencephalogram (EEG) suggests severe, diffuse cortical damage. The
patient's husband asks you if she is brain dead. The most appropriate response
is to
A. explain that it is too early to predict brain death by the legal definition
B. explain that the presence of brain stem function and posturing rules out
brain death, but the exam findings and supportive data suggest extensive
brain damage
C. inform the husband that the decision on brain death must await the
completion of a magnetic resonance image (MRI)
D. make clear that the suggestion of severe cortical damage by the EEG
implies brain death
2
E. state that the decision of brain death can only legally be made by a
neurologist
Explanation:
The correct answer is B. As part of the widely accepted University of Pittsburgh
criteria for brain death, the presence of either posturing or brain stem function
(e.g., pupillary reflexes or corneal reflexes), as is present in this case, violates
the brain criteria for the formal definition of brain death. That said, the fact that
the patient has no purposeful activity one week after an anoxic brain injury
bodes poorly for a meaningful neurological recovery.
Although stating that more time is required to pronounce brain death (choice A),
the passing of time often aids in the prognosticating of likely neurological
recovery, but the diagnosis of brain death can be made at any time and is not
time-dependent
MRI (choice C) may help assess the extent of brain injury but has no role in the
formal diagnosis of brain death.
The EEG, even when suggestive of minimal or no cortical function (choice D),
does not exclude brainstem activity and therefore cannot be used in isolation to
make the diagnosis of brain death.
Neurologists are often asked in consultation to help predict neurological
recovery and diagnose brain death (choice E) but any physician who is trained
to diagnose brain death (e.g. critical care physicians) may do so.
A 29-year-old man is brought to the emergency department by his partner
because of a headache, sluggish mentation, and impaired ambulation that has
been worsening over the past 4 days. He is HIV seropositive, but has done well
in the past and has not sought regular medical attention. During the examination
you note that his responses are slow and he has some difficulty sustaining
attention. He has a left hemiparesis with increased reflexes on the left side.
Complete blood count and electrolytes are normal. The most appropriate next
step is to
A. get a head CT with contrast
B. get a non-contrast head CT
C. perform a lumbar puncture
D. start antiretroviral therapy
E. start the patient on intravenous heparin
Explanation:
The correct answer is A. The differential diagnosis is rather broad at this point.
You should look for an infectious or malignant mass with a contrast enhanced
CT or MRI.
A non-contrast head CT (choice B) is less sensitive for abscess or tumor.
3
A lumbar puncture (choice C) should only be done after you are sure that there
is not significant mass effect.
This patient has an acute problem which should be addressed now.
Antiretroviral therapy (choice D) will help him in the long term, but does not need
to be initiated in the emergency department.
Intravenous heparin (choice E) is a treatment for embolic stroke. Embolic stroke
is unlikely in this case and he needs further evaluation before considering
treatment with intravenous heparin.
You are working in the emergency department and see a 28-year-old previously
healthy woman with a recent upper respiratory infection who has generalized
weakness, headache, and blurry vision. For the past 2 weeks she has had upper
respiratory symptoms that started with a sore throat, nasal congestion, and
excessive coughing. She went to your partner 4 days ago and was diagnosed
with sinusitis. She was given a prescription for an antibiotic and took it for 2 days
and stopped. After that she developed chills, a headache, lightheadedness,
vomiting, blurry vision, and general "achiness." The blurry vision remains when
she closes either eye. She has no drug allergies. Her temperature is 39.2 C
(102.6 F). She appears lethargic, has eye tenderness with movement, mild
photosensitivity, and nuchal rigidity. Ophthalmologic examination is
unremarkable. The most appropriate next step is to
A. get an MRI of the head, then perform a lumbar puncture
B. give her a prescription for oral azithromycin and let her go home
C. immediately administer intravenous ceftriaxone
D. immediately start intravenous acyclovir
E. obtain cerebrospinal fluid and blood cultures and observe her until the
results come back
Explanation:
The correct answer is C. The immediate concern is that she has bacterial
meningitis and she should be treated with intravenous ceftriaxone. A lumbar
puncture should be performed, however the treatment should not be delayed
until the results return.
Doing a lumbar puncture and blood draw to obtain cultures should be done
(choices A and E), however it can take a few days for the results to come back
and it may be too late for her by then. An MRI before the lumbar puncture is
recommended if the patient has papilledema, to rule out a mass that may
herniate if a lumbar puncture is performed. This is not necessary in this case
because the patient has a normal ophthalmologic exam.
Oral azithromycin (choice B) is not the proper treatment for bacterial meningitis.
Intravenous acyclovir (choice D) would be used to treat herpes encephalitis.
4
A 32-year-old man comes to the clinic because of a gradual, complete loss of smell over the
past 6 months. He is normally very healthy, does not take any medications, and does not
smoke cigarettes, or drink alcohol. He denies any recent history of trauma, change in
sleeping or eating habits. He says that he is generally happy and is satisfied with his job and
girlfriend. His temperature is 37 C (98.6 F). A complete physical examination is
unremarkable. Neurologic examination shows the inability to smell a freshly opened
isopropyl alcohol packet held 30 cm from his nose. He is also unable to smell a commercially
available odor-producing magic marker-like pen and is unable to detect phenyl ethyl alcohol.
A biochemical profile and thyroid function tests are normal. The most appropriate next step is
to
A. biopsy the olfactory neuroepithelium
B. prescribe a 14-day trial course of antibiotics to treat a possible sinus infection
C. reassure him and reevaluate in 1 month
D. refer him to a psychiatrist for evaluation
E. schedule a CT scan of the head
Explanation:
The correct answer is E. Olfactory dysfunction can be caused by many different conditions,
such as rhinitis and sinusitis, nasal polyps, nasal neoplasms, brain tumors, hypothyroidism,
and depression. The evaluation includes a complete physical and neurologic examination.
The neurologic examination should include tests of the perception of odorants (an odor
stimulus like alcohol). The next step is to rule out a neoplasm, a fracture, or paranasal
sinusitis with a CT scan or MRI.
A biopsy of the olfactory neuroepithelium (choice A) is sometimes considered but it should
wait until after a CT scan or MRI is performed.
Paranasal sinusitis may lead to olfactory dysfunction, but this patient has no other signs or
symptoms that are associated with a sinus infection. A CT scan can be used to diagnose an
infection, and if present, the infection should be treated with antibiotics. At this time, a 14-day
trial course of antibiotics to treat a possible sinus infection, (choice B), is inappropriate.
This patient has a serious problem that requires evaluation, and so it is incorrect to reassure
him and reevaluate him in 1 month (choice C). A CT scan or MRI is indicated at this time.
While olfactory dysfunction has been associated with depression, this patient does not seem
depressed, and it is inappropriate to refer him to a psychiatrist for evaluation, (choice D), at
this time.
A previously healthy 29-year-old woman comes to the office because of progressive
weakness and "painful burning" of the limbs over the past 6 months. She has a 2- month
history of blurry vision and eye pain that worsens with eye movement. She describes
occasional attacks of lancinating, shock-like facial pain. She is "incredibly busy" at work and
so it has been difficult to leave the office, but she realized that she has a "serious problem"
when she suddenly became unable to control her urination last week. She lives in an
apartment in a big city, and has not gone on a vacation in "years". She tells you that when
5
she does go away, her idea of relaxation is the beach, not the woods. Physical examination
shows sensory loss on the lower extremities, spasticity, hyperreflexia, and extensor plantar
responses. Funduscopic examination reveals pallor of the optic disc. Cerebrospinal fluid
analysis shows mononuclear pleocytosis and oligoclonal IgG. The most appropriate next
step is to
A. admit her to the hospital and begin ceftriaxone therapy, intravenously
B. admit her to the hospital and begin methylprednisolone therapy, intravenously
C. reassure her that this is a self-limited reaction to a virus
D. schedule her for a CT scan of the brain and spinal cord
E. schedule her for an MRI of the brain and spinal cord
Explanation:
The correct answer is E. This patient most likely has multiple sclerosis (MS), which is a
progressive demyelinating disease that typically begins in early to middle adulthood and is
characterized by chronic inflammation, demyelination, and sclerosis of the brain and spinal
cord. Lesions are called plaques and are usually present in the white matter. The clinical
manifestations include weakness, blurry vision due to optic neuritis, diplopia, trigeminal
neuralgia, sensory deficits, incontinence, and ataxia. The evaluation for MS includes an MRI
and a lumbar puncture. An MRI often shows hyperintense plaques and demyelination in the
brain and spinal cord. CSF findings include pleocytosis, an elevation of Ig, and oligoclonal
IgG. The clinical course is variable and relapsing-remitting and primary progressive MS are
two different categories of disease. Treatment includes interferon and corticosteroids. The
cause of the disease is unknown.
Intravenous ceftriaxone therapy (choice A) may be appropriate for Lyme disease with
neurologic involvement. Lyme disease is a Borrelia burgdorferi infection transmitted by the
Ixodes tick. It tends to affect individuals who go hiking, hunting, and camping in rural,
wooded areas (not at the beach). The infection typically involves the skin (erythema
migrans), and if left untreated, may go on to affect the joints, heart, and nervous system.
Facial palsies, polyneuropathies, radiculopathies, encephalopathy, and meningitis may
occur. The diagnosis is established with ELISA testing and western blotting. This patient's
history and symptoms are more consistent with multiple sclerosis than Lyme disease.
Intravenous methylprednisolone therapy (choice B), followed by oral prednisone may be
appropriate treatment for an attack of multiple sclerosis. However, an MRI should first be
done to support the diagnosis.
It is inappropriate to reassure her that her condition is a self-limited reaction to a virus
because Guillain-Barre syndrome (GBS) (choice C), not multiple sclerosis, usually follows a
viral infection. The typical presentation of GBS is ascending weakness and paralysis.
Sensation usually remains intact. Deep tendon reflexes are absent. Respiratory failure is a
complication. Cerebrospinal fluid analysis shows increased protein with no pleocytosis.
Treatment may include plasmapheresis and immunoglobulins. The patient in this case has
sensory loss, hyperreflexia, and CSF pleocytosis, which are inconsistent with GBS. It is
unclear whether MS is associated with a viral infection.
An MRI, not a CT scan (choice E), is used to evaluate patients with demyelinating disease
and suspected multiple sclerosis.
6
A 67-year-old woman has suffered a massive subarachnoid hemorrhage. The
patient was well until three days prior when she had a sudden loss of
consciousness. After emergent transport to the local hospital where her trachea
was intubated and a CT scan disclosed a large grade 4 subarachnoid bleed, she
was transferred to the intensive care unit. Over the past 72 hours her condition is
unchanged. She in nonresponsive to deep painful stimulus, there is no dolls' eye
or gag reflex, and corneal blink reflex is absent. She has had no narcotics,
analgesics, or paralytics. There is discord present within the family about whether
to withdraw care. The husband desires care to be withdrawn whereas the
children want to continue maximal supportive care. You explain that the most
important determining factor in helping to direct future care is
A. the decision of a court appointed legal guardian
B. the decision of the medical and nursing team taking care of the patient
C. the husband's wishes given that he is the health care proxy
D. the opinion of the hospital ethics committee
E. the patient's previously expressed wishes regarding life support if
known
Explanation:
The correct answer is E. In these difficult situations where the patient is not able
to make their own decisions and when there is disparate views among family
members, the most important directing factor should be what the patient would
have decided were they able to speak for themselves. Therefore, previously
expressed views on what the patient would want if faced with a similar situation
could prove invaluable in helping to direct care in the direction that the patient
would have wanted.
Court appointed legal guardians (choice A) are utilized when a patient has no
family or has family members who are not competent to help make medical
decisions on behalf of the incapacitated patient.
If the patient has a clearly expressed view, this view would supercede any
sentiments by the medical team (choice B), the husband (choice C), or a
hospital ethics committee (choice D).
A 26-year-old woman with a history of generalized tonic-clonic seizures since a
motor vehicle accident 5 years ago is brought to the emergency department
because of a seizure. She has not had a seizure in over 6 months until today,
when 1 hour ago she had a witnessed seizure lasting for 5 minutes. Her mother
tells you that it was typical of what her usual seizures looked like. She did not hit
her head. Her only medication is carbamazepine. Her temperature is 37.0 C
(98.6 F). She has a slight tremor of the upper extremities, is tangential, sleepy,
and mildly confused. There is no evidence of trauma or nuchal rigidity. She has
bilateral Babinski signs. A CT scan of the head is normal. The most appropriate
next step is to
A. check a carbamazepine level
7
B. discharge her with a prescription for oral phenytoin
C. get an MRI of the brain
D. give 2 mg of intravenous lorazepam
E. load her with intravenous phenytoin
Explanation:
The correct answer is A. This patient is postictal. Poor compliance is one of the
leading causes of "breakthrough" seizures. In this case, blood should have been
sent for a carbamazepine level almost immediately on arrival to the emergency
room.
Since she has not yet been fully evaluated for the cause of the present seizure,
there is no reason to start a new medication and she should not be discharged
yet (choice B).
A brain MRI (choice C) is unlikely to be helpful in this case.
Intravenous lorazepam (choice D) or phenytoin (choice E) would be indicated if
she were in status epilepticus. She is not seizing right now and probably could
even take oral medications if needed.
A 2-year-old girl is brought the emergency department by her parents because of a high
fever and generalized "body convulsions." The parents say that she was "pretty fussy" all
day and her temperature at that time was 37.3 C (99.2 F). Over the past few hours, she
began to "burn up," with her temperature spiking to 39.7 C (103.4 F). They put her into her
crib to go to sleep, and they heard "banging" coming from the baby monitor in their bedroom.
When they arrived in her room 10 seconds later, her entire body was "shaking". It lasted
about 2 minutes, which seemed like a "lifetime" to them, and did not recur. She was lethargic
and drowsy for 5 minutes after the "shaking". Nobody in the family has ever had a seizure
before. Her temperature is 39.3 C (102.8F). Neurologic examination is unremarkable. A
chest x-ray shows a left lower lobe consolidation. A lumbar puncture shows:
The parents are concerned about the severity of their daughter's condition and what they can
expect in the future. The most appropriate response is
A. "Luckily, you got to the hospital in time to avoid complications such as
hydrocephalus, hearing loss, speech or developmental delays, and mental retardation."
B. "Since you do not have a family history of febrile seizures, chances are that she will
go on to have idiopathic epilepsy."
C. "This episode is due to her pulmonary infection, and it will never recur."
D. "Your daughter will most likely have a complete recovery and there is only a very
small chance that she will develop epilepsy."
8
E. "Your daughter's condition is very serious and a full evaluation for epilepsy is
indicated at this time."
Explanation:
The correct answer is D. This patient most likely had a febrile seizure, which is a seizure
associated with fever without acute intracranial infection or electrolyte imbalance. The peak
occurrence is between 6 months and 5 years. These episodes last less than 15 minutes and
typically follow a rapid rise in temperature. The main priority is to diagnose the source of
infection. This patient seems to have pneumonia that is causing a high fever. There is a 3050% chance that she will have another febrile seizure. Patients with a generalized tonicclonic febrile seizure who have a normal neurologic examination and lack a family history of
epilepsy, only have a very small chance (2-3%) of progressing to epilepsy.
It is inappropriate to say, "Luckily, you got to the hospital in time to avoid complications such
as hydrocephalus, hearing loss, speech or developmental delays, and mental retardation"
(choice A). These complications are possible after meningitis, and this patient had a febrile
seizure, has normal cerebrospinal fluid, and pneumonia, which makes the diagnosis of
meningitis unlikely. Also, even if she did have meningitis, it is inappropriate to tell the parents
that they avoided these complications because there is no real way to determine that at this
time.
It is inappropriate to say, "Since you do not have a family history of febrile seizures, chances
are that she will go on to have idiopathic epilepsy" (choice B). The parents said that nobody
in the family has ever had a seizure, which most likely means a febrile seizure or epilepsy. A
lack of family history of febrile seizures (or epilepsy) does not increase a patient's risk of
developing epilepsy. If there was a family history of seizures, especially epilepsy, she would
be at an increased risk of developing epilepsy.
"This episode is due to her pulmonary infection, and it will never recur" (choice C) is incorrect
because while this febrile seizure was most likely due to the fever associated with her
pulmonary infection, you cannot say that it will "never" recur. Febrile seizures recur in 3050% of children, but only rarely progress to epilepsy.
"Your daughter's condition is very serious and a full evaluation for epilepsy is indicated at this
time" (choice E) is incorrect because she most likely had a febrile seizure, due to the rapid
rise in temperature associated with a pulmonary infection. Her lumbar puncture was
completely normal. A full evaluation for epilepsy is generally not indicated in the case of a
simple febrile seizure.
A 21-year-old right-handed university student comes to the clinic for evaluation of
a history of numbness in the left lower extremity. Two weeks ago she was
working in the photography lab and she was standing all day. After that, she
experienced a cold sensation in the left foot and her entire left leg felt "asleep". It
lasted 4 to 5 days and it slowly went away. Her right lower extremity was fine.
Coughing, sneezing, and Valsalva maneuvers did not worsen her symptoms.
She had a slight back pain but she attributes this to the poor quality mattress in
her dorm room. She tells you that she had an episode of "optic something-it is" in
the left eye 2 years earlier. At that time, she was reportedly depressed and she
was sleeping constantly. One day, her left eye became blurred and her vision
went out. In 1 week her vision returned to normal. Her vision now is 20/20. She
has not had a repeat episode since then. She had an MRI of her brain, which
was reportedly negative. She admits to drinking alcohol occasionally, but denies
9
any illicit drug use. Her only medication is birth control pills. Examination is
significant for brisk reflexes and sustained clonus at the right ankle. A Babinski
sign is present on the right. The next most appropriate action in managing this
patient is to
A. obtain an outpatient neurological consultation within 1 week
B. prescribe daily aspirin therapy
C. prescribe intravenous corticosteroid therapy
D. prescribe warfarin therapy
E. send her to the emergency department for a neurological consultation
Explanation:
The correct answer is A. This is a typical history for multiple sclerosis (MS). This
is a sometimes a difficult diagnosis to make and the management can be
complicated. The patient should be referred to a neurologist. However, it is not
emergent (choice E) in this case, particularly since her most recent episode has
resolved.
Daily aspirin (choice B) is a treatment for cerebrovascular disease. This patient's
history and examination are more consistent with multiple sclerosis than with
cerebrovascular disease.
Intravenous corticosteroids (choice C) are a treatment for acute MS attacks.
Presently, she is asymptomatic and should be sent for evaluation by a
neurologist.
Warfarin (choice D) is a treatment for cerebrovascular disease, which she
probably does not have.
A 63-year-old woman comes to the office because of a 3-week history of a "dull,
achy" headache. She says that it started out as an intermittent headache that
was exacerbated by bending down, lifting heavy objects, sneezing, defecating,
and coughing, but lately it has become constant. She cannot associate the
headaches with food or hunger, alcohol, weather or barometric pressure
changes, sounds, or irregular sleep patterns. She is generally very healthy, but
recalls having some nausea and vomiting a few weeks before the headaches
started. She has never had headaches before. She does not take any
medications, rarely drinks alcohol, and exercises regularly. Her temperature is
37.0 C (98.6 F), 130/80 mm Hg, pulse is 70/min, and respirations are 15/min.
Physical examination is unremarkable. An erythrocyte sedimentation rate,
complete blood count, and electrolytes are normal. A trial of oral prednisone,
sublingual ergotamine, and oral sumatriptan is ineffective. The most appropriate
next step is to
A. administer glucocorticoids, intravenously
B. administer sumatriptan, intramuscularly
C. give her oxygen inhalation therapy
10
D. obtain a temporal artery biopsy
E. order an MRI of the head
F. perform a lumbar puncture
Explanation:
The correct answer is E. In any patient with a new headache that is associated
with nausea and vomiting and exacerbated by exertion or positional changes, a
brain tumor should be suspected. This is the typical presentation of a posterior
fossa brain tumor. Weight loss, systemic symptoms, and an elevated
erythrocyte sedimentation rate are not typically associated with a primary brain
tumor. Papilledema, caused by increased intracranial pressure, is rarely found in
patients over 55 years old with an intracranial mass. An MRI or CT scan of the
head will establish the diagnosis.
Administration of glucocorticoids, intravenously (choice A) is the appropriate
treatment to prevent blindness in a suspected case of temporal arteritis.
Temporal arteritis typically presents with a unilateral headache, myalgias, jaw
pain, fever, and weight loss. A tender, reddened temporal artery is often found.
Visual changes may occur and blindness is a feared complication. This patient
does not have any of the usual systemic symptoms associated with temporal
arteritis. The diagnosis of temporal arteritis is established with a temporal artery
biopsy (choice D). The erythrocyte sedimentation rate is typically elevated in
temporal arteritis
Administration of sumatriptan, intramuscularly (choice B) is the treatment for a
migraine headache which typically presents with a throbbing headache, nausea,
vomiting, photophobia, and functional impairment. An aura consisting of
hallucinations and scotomas sometimes occurs before the headache. An
intermittent headache that becomes constant is not the typical pattern of a
migraine. The sublingual ergotamine and oral sumatriptan that this patient was
taking are typically effective in treating a migraine.
Oxygen inhalation therapy (choice C) is the treatment for a cluster headache. A
cluster headache is an episodic headache that typically presents with a few
short headaches a day for a few weeks that is associated with periorbital pain,
reddening of the eye, and lacrimation. This patient's history is inconsistent with a
cluster headache.
A lumbar puncture (choice F) should be performed in a suspected case of
meningitis and possibly in a suspected intracranial hemorrhage, if the CT scan
fails to show the bleed. Meningitis often presents with a headache, nuchal
rigidity, and photophobia. Fever and a rash may be present. An intracranial
hemorrhage may present with a headache and nuchal rigidity, seizures, and
confusion. A CT scan of the head usually establishes the diagnosis. The
patient's history is inconsistent with meningitis and is more consistent with a
brain tumor than an intracranial bleed. A lumbar puncture should not be
performed in this patient or any patient with a suspected brain tumor because it
may cause brain herniation.
11
A 28-year-old woman with AIDS is admitted to the hospital because of confusion
and difficulty walking. Her blood pressure of 130/80 mm Hg and pulse is 104/min.
She is aphasic and has a moderate right hemiparesis. Her chart indicates that
she has hepatitis C and recently suffered from Pneumocystis carinii pneumonia
(PCP). A CT scan of the head shows several rim-enhancing lesions. Serologic
studies demonstrate the presence of IgG antibodies to Toxoplasma gondii. Her
CD4 lymphocyte count is 50/mm3. Appropriate treatment for this patient should
include
A. intravenous acyclovir
B. neurosurgical removal of the lesions
C. oral fluconazole
D. sulfadiazine and pyrimethamine
E. thiabendazole
Explanation:
The correct answer is D. Sulfadiazine and pyrimethamine is proper treatment for
T. gondii infection.
Intravenous acyclovir (choice A) is used to treat herpes encephalitis.
Neurosurgical removal of the lesions (choice B) is not indicated.
Oral fluconazole (choice C) is a treatment for fungal infections.
Thiabendazole (choice E) is used to treat helminth infections.
You are seeing a 34-year-old woman in the emergency department who is now
10 days post emergent C-section for preeclampsia at 29 weeks, and presents
with a 3-day history of a headache. She describes it as "pain starting at the right
temple and shooting through the head to the left temple". There is some
associated photophobia and phonophobia. At its worst, the headache is a 10 of
10 on the pain scale, with 10 being the worst pain that she ever felt in her life.
This is similar to a headache she was having a month ago during her pregnancy.
There is no relief with acetaminophen. Prior to delivery she had numbness of her
right cheek that was transient. Her blood pressure is 160/90 mm Hg. Examination
shows bilateral papilledema. The most appropriate next step in management is to
A. begin intravenous heparin
B. call a neurosurgical consult
C. discharge the patient with a prescription for naratriptan
D. discharge the patient with a soft cervical collar and prescription for
metaxalone
E. order a brain MRI and MRV
12
Explanation:
The correct answer is E. This presentation is concerning for venous sinus
thrombosis. MRV is the best noninvasive test to confirm this diagnosis. If the
MRI/MRV is negative, then she should have a lumbar puncture to rule out
idiopathic intracranial hypertension (pseudotumor cerebri).
Intravenous heparin (choice A) is the usual treatment for a venous sinus
thrombosis, however, the diagnosis should be confirmed first if possible.
Therefore, a brain MRI and MRV is the most appropriate next step in
management.
A neurosurgical consult (choice B) is not indicated in this case.
Naratriptan (choice C) is used for the acute treatment of migraine headaches.
This is atypical for a migraine, but she should be further evaluated before she is
discharged and treated.
Soft cervical collar and metaxalone (choice D) are used to treat headaches from
cervical muscle spasms. There was no neck muscle spasms noted on exam,
and she should be further evaluated before she is discharged.
An 18-year-old woman with no past medical history, no allergies, and taking no
routine medications comes to the local college clinic for headaches and lowgrade fevers. She is discharged home with the diagnosis of a "viral syndrome"
and instructed to get ample rest. Approximately three hours later her roommate
calls 911 reporting that her friend is unconscious and not arousable. On arrival,
paramedics find a lethargic, febrile, unresponsive woman lying on the floor. Her
pupils are dilated but responsive and she has a petechial and purpuric rash over
70-80% of her body area. They bring her to the emergency department. The
most likely organism responsible for this patient's condition is
A. Escherichia coli
B. Haemophilus influenza
C. Listeria monocytogenes
D. Neisseria meningitidis
E. Streptococcus pneumoniae
Explanation:
The correct answer is D. This patient with altered mental status, a rapid
neurological decline, and skin lesions all point to fulminant infection with
Neisseria meningitidis.
E. coli (choice A) is generally not a disease that, when it becomes blood-born,
produces anything other than the sepsis syndrome. A petechial or purpuric rash
is not such a feature.
The same holds true for Haemophilus influenza (choice B) that is most often
seen in young children, but has been decreasing in incidence since wide-spread
13
effective vaccination programs have been implemented.
Listeria monocytogenes (choice C) is often a disease of newborns and neonates
and again, does not present with characteristic skin lesions.
Streptococcus pneumoniae meningitis (choice E) is a common etiologic agent
for pneumonia in adults. It can also be associated with significant morbidity and
mortality but is not associated with a rapid (<12 hour) neurological decline, nor
with purpuric skin lesions. The skin lesions on this patient are believed to be the
result of an immune complex deposition in the skin capillaries.
A 72-year-old man comes to the clinic because of a 4-month history of
intermittent headaches. He has no prior history of significant headaches. He
describes the pain as unilateral, always on the right, generally located in the
temporal region. On his review of systems, he complains of some pain and
stiffness in his neck, shoulders, and hips, which has also come on in the past few
months. Furthermore, he has noticed an unusual pain and tiredness in his jaw
near the end of meals. His temperature is 37.2 C (99 F), blood pressure is
122/78 mm Hg, pulse is 62/min, and respirations are 16/min. His pupils are
equally round and reactive to light, and his extraocular movements are normal.
Visual acuity is 20/30, which is stable for him. The remainder of his neurologic
examination is unremarkable. Laboratory studies show an erythrocyte
sedimentation rate (ESR) of 124 seconds. Treatment should be instituted
immediately to prevent
A. death from further intracranial hemorrhage
B. death from metastatic tumor
C. death from uncal herniation due to mass effect
D. progressive visual loss
E. severe morbidity from a large embolic stroke
Explanation:
The correct answer is D. This patient is presenting with signs and symptoms of
temporal arteritis, also known as giant cell arteritis. This disease is caused by
inflammation of the large vessels off of the aortic arch, most commonly
manifesting in the temporal artery. Biopsy will show mononuclear and giant
multinucleated cells in the intima and media of the vessel. Classic symptoms are
new onset temporal headaches in a patient over 60 years of age, jaw
claudication as he describes, and an ESR that is markedly elevated. Patients
often have pain to palpation over the temporal artery itself. There is also a high
association with polymyalgia rheumatica (PMR), which causes limb girdle pain
and stiffness as this patient describes. Progressive visual loss, due to the effect
of the disease on the ophthalmic artery can befall up to 50%, in untreated
patients.
Intracranial hemorrhage (choice A) is also very unlikely in this case. Longer-term
headache can be seen with chronic subdural hematomas. However, this patient
14
also has jaw claudication, symptoms of PMR, and an elevated ESR, which go
along more with temporal arteritis.
Death from metastatic tumor (choice B) or herniation of the uncal region of the
brain (choice C) would be of concern in the setting of a brain tumor. Although
headache can be an initial symptom in patients with central nervous system
malignancies, it is not a likely consideration in this case.
There is nothing to suggest from the case presentation that he is at high risk for
an embolic stroke (choice E).
A 17-year-old obese girl comes to the office because of a bifrontal, non-throbbing
headache that has been worsening over the past 6 months. It often wakes her
from sleep, but improves if she gets up and walks around. She also says that her
vision is "not as good as it used to be," but cannot give you more details.
Examination is significant for mild optic disk blurring. She went to the local
emergency department 5 days earlier with similar complaints and an MRI of the
head was performed and reported as normal. The most appropriate next step in
managing this case is to
A. obtain an electroencephalogram
B. obtain a neurosurgical consult
C. perform a lumbar puncture
D. repeat the MRI of the head
E. start propranolol
Explanation:
The correct answer is C. The history and exam are typical for idiopathic
intracranial hypertension (pseudotumor cerebri). The exact cause is unknown,
but it is probably due to impaired reabsorption of cerebrospinal fluid leading to
raised intracranial pressure. It usually occurs in obese, young women. Lumbar
puncture shows an elevated opening pressure and normal CSF. Acetazolamide
may be used to lower intracranial pressure. Weight reduction is important.
Obtaining an electroencephalogram (choice A) will not help with the diagnosis.
There is no reason for a neurosurgical consult (choice B).
Since nothing has changed since last week and she has the exact same
symptoms, a repeat MRI of the head (choice D) is not indicated. It would be
appropriate if she had different symptoms.
Propranolol (choice E) is used for migraine prophylaxis and is not indicated
here, as this patient's condition is consistent with idiopathic intracranial
hypertension, and further evaluation is indicated.
A 45-year-old woman collapses while in her bathroom and is brought to the
15
hospital emergently. On arrival, she is non-responsive with dilated and fixed
pupils bilaterally. Her temperature is 37.2 C (99 F), blood pressure is 220/110
mm Hg, pulse is 42/min, and respirations are 18/min. An emergent CT scan of
the head reveals blood in all 4 ventricles. An intracranial bolt is placed for
intracranial pressure (ICP) monitoring and she is brought to the operating room
for decompression craniotomy. After the procedure, she is brought to the
intensive care unit where she does well for 48 hours. Her ICP begins to fall and
she shows some signs of neurological recovery. On postoperative day number 2
her mental status again deteriorates. There is no new blood present on CT scan.
The most appropriate management is to
A. administer nimodipine, intravenously
B. administer pressors, intravenously
C. administer verapamil, intravenously
D. provide no therapy at this time
E. return her to the operating room
Explanation:
The correct answer is A. This patient has suffered a devastating subarachnoid
bleed. The most common non-traumatic cause of SAH is rupture of a preexisting
aneurysm. The mortality rate of these bleeds is high, but in managing these
patients much has been learned about how to minimize morbidity in the first
post-bleed weeks. This patient had a vasospasm. This is a nearly universal
event after a SAH since the blood in the CSF is an irritant. The treatment shown
to have fairly good benefit is the calcium channel blocker nimodipine. No other
blocker, including verapamil (choice C), has been shown to be beneficial.
Starting intravenous pressors (choice B) in these patients is the next
management step after nimodipine. This is done to maximize cerebral perfusion
pressure (mean arterial pressure minus ICP) in an attempt to maintain cerebral
perfusion.
Not more than a few years ago, vasospasm was not treated since, many
thought that the deterioration after bleeding was the natural history of the
disease. Not initiating therapy (choice D) is no longer the standard of care.
Since there is no sign of new blood on the CT scan, there is no reason to return
to the operating room (choice E).
A 43-year-old woman comes to the emergency department because of a headache, which
she says "feels like a knife stabbing starting on the left side of the lip and radiating up to the
temple". As the headache worsens, she also gets a numbness or "lack of feeling" in her left
lip, nose, and eye. The pain is constant and her left face and head are very sensitive to
touch. The headache fluctuates in intensity, but has been present since about 2 weeks ago.
No matter what she does, the pain is there. When it is bad she cannot sleep and when she is
able to fall asleep, it usually wakes her up. She sometimes gets watery eyes and pain in her
left eye with movement. She saw an otolaryngologist who told her that it is not a sinus
problem. Three or four months ago she was diagnosed with shingles of her left face and
16
head. However, there was never any skin manifestations and she was diagnosed by the
symptom of pain and reportedly positive serology. The pain at that time was similar to the
present pain, but it seemed to radiate from her left frontal-parietal area toward the lip and
now it is the reverse. She had a CT scan of the head 1 week ago, which she was told was
normal. She did not get very many headaches at all prior to this. She has been treated for
depression, sinus problems, including surgery 8 years ago, and receives cosmetic botulinum
toxin injections. Examination is significant for sensation to light touch abnormal and pin prick
decreased in the left V1 & V2 distribution. Corneal reflex in left eye is decreased in the
superior left cornea compared with the inferior left cornea. Masseter contraction is decreased
on the left. The most appropriate next step is to
A. call a neurology consult and order an MRI
B. call a neurosurgical consult
C. give intravenous prochlorperazine
D. prescribe a soft cervical collar and metaxalone
E. prescribe zolmitriptan
Explanation:
The correct answer is A. This patient has some type of facial pain syndrome. Since the pain
is constantly present, it is not a typical trigeminal neuralgia. The patient should have a MRI
and be evaluated by a neurologist.
A neurosurgical consult (choice B) is not indicated at this time. Patients with facial pain which
cannot be controlled medically or have a clear surgical etiology should be seen by a
neurosurgeon.
Intravenous prochlorperazine (choice C) and zolmitriptan (choice E) are used for the acute
treatment of migraine headaches. This patient does not describe the typical migraine
headache, which is throbbing and worsened by light, certain foods, and other factors. It may
also be preceded by an aura.
Soft cervical collar and metaxalone (choice D) are used to treat headaches from cervical
muscle spasms. This patient most likely has a facial pain syndrome that should be evaluated
by a neurologist.
A 66-year-old man with type II diabetes mellitus and atrial fibrillation comes to the
emergency department with right body weakness and slurred speech that he
noticed upon awakening in the morning. There were no complaints of word
finding difficulties and no dysesthesia. He smokes a pack of cigarettes a day and
"rarely exercises." His wife hands you a prescription medicine bottle of warfarin
and tells you that he has been taking this "for some time now." His blood
pressure is 210/95 mm Hg and his pulse is irregularly irregular. He has left-sided
neglect with slurred speech and weakness of the right body; face and upper
extremity worse than lower extremity. Routine chemistries and cell counts are
normal. His INR is 1.7. A CT scan of the head shows a large left-sided subdural
hematoma. The most appropriate next step is to
A. administer fresh frozen plasma and vitamin K
B. give her intravenous labetalol immediately
17
C. order a brain MRI
D. start him on heparin
E. tell the family that he will die and do nothing further
Explanation:
The correct answer is A. You should reverse the warfarin. The risk of him having
a stroke from atrial fibrillation is far outweighed by the immediate goal of
stopping the intracranial bleeding.
This patient requires a high blood pressure to maintain cerebral perfusion. If you
drop his blood pressure with labetalol (choice B) this will cause his brain to lose
oxygen. Also, subdural bleeds are venous, so the high arterial blood pressure
would not be expected to increase the bleeding.
A brain MRI (choice C) will not provide additional useful information at this time.
Heparin (choice D) is contraindicated during an intracranial bleed. The goal is to
reverse anticoagulation.
It is too early to predict outcome. You should not tell the family that he will die
and do nothing further (choice E).
A 41-year-old-man comes to the clinic because of right facial weakness for the
past 6 days. He denies any pain or changes in hearing. The patient visited the
office last week because of an upper respiratory infection for which you advised
symptomatic therapy. He has no significant past medical history and takes no
medications. Vital signs are: temperature 37 C (98.6 F), blood pressure 90/70
mm Hg, pulse 90/min, and respirations 15/min. Oxygen saturation is 99% on
room air. Physical examination reveals weakness of the right side of the face with
a droop. The left side of the face is normal. The remainder of a complete
neurologic examination and a mini-mental status examination are normal. The
next step in the management of this patient is to
A. administer prednisolone intravenous pulse therapy
B. advise him to take aspirin
C. order a CT scan of the head
D. order an MRI of the brain
E. order a nerve conduction study
F. prescribe ampicillin
G. provide reassurance and close clinical follow up
Explanation:
The correct answer is G. This patient has Bell's palsy, an idiopathic
inflammatory neuropathy of the facial nerve usually following a viral upper
18
respiratory infection. Over 90% of patients recover without residual symptoms in
1 month. Oral prednisone with or without acyclovir is sometimes used to shorten
the recovery period and possibly improve the outcome. However, this is based
on a limited amount of studies.
Prednisolone therapy (choice A) is not indicated. High-potency intravenous
steroids are not used in Bell's palsy as they have no proven benefit and many
side effects. Low-potency oral steroids are used anecdotally, although they have
no proven benefit.
Aspirin (choice B) has no direct role in the treatment of Bell's palsy.
A CT of the head (choice C) is not necessary in the evaluation of Bell's palsy.
Focal facial nerve neuropathy following an upper respiratory infection is the
classic presentation of Bell's palsy. The presence of other neurologic symptoms
or signs would point to a primary neurologic process and would then warrant
some type of neuroimaging.
An MRI of the brain (choice D) is not necessary in the evaluation of Bell's palsy.
Focal facial nerve neuropathy following an upper respiratory infection is the
classic presentation of Bell's palsy. The presence of other neurologic symptoms
or signs would point to a primary neurologic process and would then warrant
some type of neuroimaging.
Nerve conduction studies (choice E) are not necessary in the evaluation of Bell's
palsy.
Ampicillin (choice F) is not indicated because a facial nerve neuropathy is not
due to a bacterial infection. The patient should be managed conservatively for
Bell's palsy.
A 36-year-old woman comes to the office complaining of double vision. She says
that for the last few weeks she has been feeling "weak all over," especially at the
end of the day, which is also when she thinks her double vision is most
noticeable to her. She reports that she experienced similar symptoms 1 year ago
that persisted for several weeks, but she did not seek a medical opinion at that
time because she thought that it was "just stress." She has no other complaints.
She has no past medical history. Her mother has Graves disease, her father has
rheumatoid arthritis, and her older sister has been recently diagnosed with
systemic lupus erythematosus. On examination, she is a slim woman who
appears in good health. Her vital signs are normal. She has normal heart and
lung sounds. She has mild bilateral ptosis and complains of diplopia when her
extraocular muscles are tested. She has normal strength at maximal effort, but
her proximal muscles are fatigable. A complete blood count, electrolytes, and
thyroid-stimulating hormone levels are unremarkable. Antinuclear antigen, antiDNA antibody, and rheumatoid factor are negative. Acetylcholine receptor
antibody test is positive. You arrange for an electromyelogram, which shows a
decremental response to repeated nerve stimulation. The most appropriate next
step is to order a
A. CT scan of the chest
19
B. lumbar puncture
C. MRI of the brain
D. muscle biopsy
E. Tensilon test
Explanation:
The correct answer is A. The patient has myasthenia gravis, which is
characterized by fluctuating weakness in a characteristic distribution, usually
causing diplopia, ptosis, and proximal muscle weakness. It is caused by an
autoimmune response that produces an antibody to acetylcholine receptor. The
diagnosis is based on clinical suspicion and confirmed by the acetylcholine
receptor antibody test or by an EMG that shows decremental response to
repetitive stimulation. One can also perform a single fiber EMG, which is
possibly the most specific test, but not required for the diagnosis. In patients
with confirmed MG, it is important to look for other autoimmune disease such as
RA, SLE, thyroiditis, and Graves disease, which have a higher incidence in
patients with MG and their first degree relatives. After the diagnosis is
confirmed, it is important to look for evidence of thymoma, especially in patients
younger than age 60. The reason is that thymectomy is recommended for most
patients with MG, the best responses to it are in young patients with evidence of
thymoma. The response to thymectomy is variable and may not be noticed until
2-5 years after the surgery.
Lumbar puncture (choice B) is incorrect. Based on the clinical history, physical
exam, and labs, the patient has myasthenia gravis and there is no role for
examining the cerebrospinal fluid.
MRI (choice C) is incorrect. MRI of the brain is a good test to evaluate for
structural abnormalities of the central nervous system. Myasthenia gravis does
not affect the central nervous system and therefore, there is no role for MRI.
Muscle biopsy (choice D) is incorrect. The patient has complaints of weakness
that are characteristic for myasthenia gravis. However, myopathy should remain
in the differential diagnosis until confirmatory tests are done. In this case, the
labs and EMG did confirm MG. If they did not confirm MG and one wanted to
pursue a diagnosis of myopathy, muscle biopsy would still not be the first test to
perform. Instead, one would begin with serum CK and aldolase levels.
Tensilon test (choice E) is incorrect. A Tensilon test is a simple but subjective
test that may be performed in the office or at the bedside. It may be used to
clinically corroborate a suspected diagnosis of MG, but may be positive in
diseases other than MG, such as motor neuron disease. Thus, it does not
replace the acetylcholine receptor antibody test or the EMG in the diagnosis of
MG. Furthermore, in this case, MG has already been diagnosed by the above
tests and no additional information will be obtained by doing a Tensilon test.
A 69-year-old man is admitted to the hospital because of palpitations. He has a
long history of mitral stenosis secondary to rheumatic disease. The patient
awoke this morning with mild chest discomfort and a feeling that his "heart was
20
racing in his chest." His family called an ambulance and he was taken to the
emergency department where he was found to be in atrial fibrillation at a rate of
160/min. He was given a beta-blocker and started on digoxin therapy. This
lowered his heart rate to 110/min and the symptoms resolved. He is admitted to
the hospital for further management. On the floor, he is started on intravenous
unfractionated heparin and is scheduled to undergo a transesophageal
echocardiogram in the morning. On discussing the implications of atrial fibrillation
with the patient, the team focuses on the long-term risks of cerebrovascular
accident. The most accurate statement concerning the patient's risk of a
cerebrovascular accident is:
A. It is dependent on the presence of coronary artery disease
B. It is greater than the general population, but less than a patient with
atrial fibrillation only
C. It is greater than the general population, but less than a patient with
congestive heart failure and atrial fibrillation
D. It is greatest of any population
E. It is the same as the general population
Explanation:
The correct answer is D. A number of studies have conclusively established that
the risk of stroke for patients over the age of 65 with atrial fibrillation (AF) is
significant and warrants therapy in all cases. The magnitude of this risk depends
on the underlying etiology. Most patients with AF have a significantly increased
risk of embolic stroke. Patients with rheumatic heart disease (RHD) have a 17fold higher risk than the general population and patients with non-RHD have an
estimated 5-fold increased risk.
The risk of stroke depends on the presence of coronary artery disease (choice
A) is true, but only for non-RHD patients. The risk for these patients is not
uniform and is instead based on the presence or absence of a number of risk
factors, coronary artery disease being one of them. However, for RHD patients,
the risk for stroke is almost uniformly determined by the magnitude of the
valvular lesion and the size of the atrium.
The risk of stroke is greater than the general population, but less than with lone
atrial fibrillation (choice B) is incorrect. Lone AF is AF with no underlying risk
factors, such that occurs after cardiac surgery or below the age of 65. These
patients have a very low annual risk for embolic events. These patients are
usually treated with aspirin.
The risk of stroke is greater than the general population, but less than with
congestive heart failure and atrial fibrillation (choice C) comprises the so-called
non-RHD group of AF patients, and as mentioned above, has a 5-fold increased
risk of stroke compared to 17-fold for RHD patients.
The risk for stroke is not different from the general population (choice E) is
clearly incorrect because it is the greatest of any population.
21
A 14-year-old boy is brought to the emergency department after he collapsed on the high
school football field during a game. He is awake and alert but is unable to tell you exactly
what happened. His teammate says that the patient was running down the field and was
about to catch the ball, but he collapsed before he even made contact with the ball. The
patient remembers becoming dizzy before he collapsed, but he did not experience any arrest
of motion during this time. He has no medical conditions and does not take any medications.
He is sexually active with 2 different girls and they "sometimes" use condoms for protection.
His temperature is 36.7 C (98.0 F), blood pressure is 110/70 mm Hg, pulse is 100/min, and
respirations are 14/min. Physical examination is normal. Blood is drawn and sent for
evaluation of electrolytes, BUN, creatinine, magnesium, and calcium. The most appropriate
next step is to
A. admit him for cardiac monitoring
B. order echocardiography
C. order electrocardiography
D. order electroencephalography
E. request consultation with a neurologist
F. schedule a tilt-table test
Explanation:
The correct answer is C. This young, healthy, athletic patient had a syncopal episode, which
can be cardiogenic or neurogenic in origin. After the history and physical, blood work is
usually sent to rule out anemia, infections, hypocalcemia, or hypomagnesemia and an EKG
must be performed. An EKG may show evidence of cardiac abnormalities such as WolfParkinson-White syndrome (r wave slurring), idiopathic hypertrophic subaortic stenosis, or
congenital prolonged QT syndrome.
It is inappropriate to admit him for cardiac monitoring (choice A) before even performing an
EKG. You need to first try to distinguish the etiology of this syncopal episode, therefore an
EKG and other studies should be done in the emergency department. Cardiac monitoring
may be necessary if an arrhythmia is found.
An echocardiogram (choice B) may be necessary to evaluate cardiac abnormalities,
however this should not be done until an EKG is performed.
An electroencephalogram (choice D) may be performed if a seizure is suspected. However
this should not be performed at this time in this patient because it does not seem like he
had a seizure.
A neurology consultation (choice E) is not necessary at this time because you must first try
to determine whether this syncopal episode was cardiogenic or neurogenic in etiology. An
electrocardiogram is an easy test to perform in the emergency department to assess for
cardiac abnormalities.
A tilt-table test (choice F) is useful in establishing the diagnosis of
vasovagal/neurocardiogenic syncope, which is caused by venous pooling, decreased
venous return, inappropriate vasodilatation, hypotension, and relative bradycardia. A
positive test will show hypotension and bradycardia, and syncope. This test is not part of the
initial evaluation of a single syncopal episode in a young, healthy, athletic patient. It is
22
usually used in patients with multiple unexplained syncopal episodes. An electrocardiogram
is easier to perform and is more likely to help establish a diagnosis in this patient at this
time.
A 40-year-old man with hypertension comes to the emergency department
complaining of a severe headache that started the previous evening while
exercising at the gym. He states that the headache came on suddenly and
describes a constant throbbing pain on the top of his head and down the back of
his neck. He did not experience any loss of consciousness, but does complain of
nausea and says that he vomited twice last night. He also complains of blurry
vision, dizziness, and fatigue. He experienced a similar, but less severe
headache one week ago that resolved spontaneously after about 2 hours. He
has tried ibuprofen and some acetaminophen with codeine that he had left over
from a previous tooth extraction, but has not had any relief of his symptoms. He
waited to come to the hospital this morning because he thought the headache
was due to the stress he has been experiencing at work and that it would resolve
on its own. He smokes 1 pack of cigarettes per day, drinks alcohol occasionally,
and does not do any illicit drugs. His temperature is 37.0 C (98.6 F), blood
pressure is 158/90 mm Hg, pulse is 62/min, respirations are 18/min, and oxygen
saturation is 99%. Physical examination shows a well-developed man lying on
the stretcher with his right hand covering his eyes in obvious distress secondary
to pain. His pupils are equal, round, and reactive to light and accommodation
however, you are unable to examine of the fundi due to the patient's discomfort
with the light. Cranial nerves II-XII are intact. He has some neck stiffness with
flexion and extension. The remainder of the physical and neurologic examination
shows difficulty with bilateral finger to nose precision as well as heel to shin
coordination. You order a non-contrast CT scan of his head, which is read as
normal. The most appropriate next step in the management of this patient is to
A. administer intravenous analgesia with morphine and metoclopramide
and observe in the emergency department
B. administer subcutaneous sumatriptan and discharge home to follow up
with a neurologist
C. obtain an urgent neurosurgical consultation and a cerebral angiogram
D. perform a lumbar puncture with CSF analysis
E. request psychiatric evaluation for acute stress reaction
Explanation:
The correct answer is D. In this patient, you should perform lumbar puncture
with CSF analysis for suspected subarachnoid hemorrhage (SAH). The CT scan
can be normal in 5% of patients with a subarachnoid hemorrhage within the first
12 hours of headache onset. Between 24 and 72 hours from onset of headache,
up to 25% of CT scans can be normal. Therefore, if one suspects a
subarachnoid hemorrhage from the patient's history, it is imperative to perform a
lumbar puncture to evaluate the CSF for xanthochromia and red blood cells.
Xanthochromia is not seen in a traumatic tap and may be more specific than
RBC analysis. A lumbar puncture will also help to rule out meningitis, although
this is unlikely in this patient given his presentation.
23
Giving pain medication (choice A) is acceptable, but the patient should have a
more thorough evaluation.
Giving a 5-HT agonist (choice B) is a bad idea in this patient because it may
lead to vascular spasm and worsen the subarachnoid bleeding.
Neurosurgical evaluation and cerebral angiography (choice C) may be
necessary, but further evaluation is needed to justify this consult.
Prior to psychiatric evaluation (choice E), one must first rule out life threatening
causes of headache.
A 58-year-old left-handed man comes to the clinic because of a 7-month history
of "left hand involuntary twitches." His hand typically shakes when he is at rest
and he can usually stop the shaking by looking at his hand and concentrating.
The shaking does not impair his activities in any way and he has no trouble
holding a glass of water. There is no tremor in his right hand and the lower
extremities are not affected. He has had no trouble walking and there have been
no behavioral or language changes. On examination, a left hand tremor is
evident when he is distracted. Handwriting is mildly tremulous. He has mild
bilateral cogwheel rigidity with contralateral activation. He is not bradykinetic. The
most appropriate next step would be to
A. order electromyography and nerve conduction studies
B. prescribe carbidopa/levodopa
C. prescribe propranolol
D. schedule an outpatient neurological consultation
E. send him to the emergency department for neurological evaluation
Explanation:
The correct answer is D. The tremor is of a Parkinsonian type and he should be
examined by a neurologist.
Electromyography and nerve conduction studies would not be useful in
diagnosing the tremor (choice A).
His Parkinsonian features are very mild and do not interfere with his daily
activities, so treatment with carbidopa/levodopa (choice B) is not indicated at
this time.
Propranolol (choice C) may be effective for kinetic and postural tremors, but is
not a treatment for a Parkinsonian type tremor.
This patient has a Parkinsonian tremor that should be evaluated by a
neurologist, but there is nothing to suggest that it be done emergently (choice
E).
24
A 39-year-old woman who had a hysterectomy and bilateral oophorectomy 5
months earlier for heavy irregular menses comes to the office because of
"strange feelings." She describes a "rising discomfort in her stomach followed
immediately by feelings of rage." The episodes usually last a few minutes. She
has had them in the past, but since the surgery they have increased in
frequency. She has no chronic medical conditions that she is aware of and is
taking estrogen replacement therapy. Physical examination is normal. The most
appropriate next step is to
A. immediately send the patient to the emergency department
B. obtain a neurological consult
C. obtain a psychiatric consult
D. order a brain MRI
E. start the patient on phenytoin and see her back in 6 months
Explanation:
The correct answer is B. This patient gives a typical history for temporal lobe
epilepsy. Seizures are often exacerbated by estrogen, which is a
neuroexcitatory hormone. The neurological examination, including mental status
exam, is often normal in these cases. This is a complicated case and should be
managed at least initially by a neurologist.
There is no acute issue that would warrant sending this patient to the
emergency department (choice A).
There is no history suggestive of mental illness. A psychological etiology for her
symptoms must be a diagnosis of exclusion, so a psychiatric consult is not
necessary (choice C).
A brain MRI (choice D) might detect an abnormality contributing to her seizures.
However, a neurologist would probably order specific MRI sequences based on
his evaluation and the test would need to be repeated anyways. Additionally, the
neurologist may want to order other tests.
A rising discomfort in the stomach followed immediately by feelings of rage is
highly suggestive of temporal lobe epilepsy. However, further evaluation should
be done before starting phenytoin (choice E). Additionally, she will need to be
closely followed and phenytoin is often a poor first choice anti-seizure
medication for a young woman.
An 82-year-old man is admitted to the hospital after he started dragging his right
leg today. Workers at the nursing home where he resides also noticed that he
was not as talkative as usual. His temperature is 37.0 C (98.6 F). His speech
pattern is non-fluent. On the right side, hip flexion, knee flexion, dorsiflexion, and
abductor hallucis longus were weaker than other lower extremity muscles. Cell
counts, chemistries, and coagulation studies are normal. A CT scan of the head
shows a 1.5 cm left-sided subdural hematoma. His granddaughter arrives after
he returns to his room from the CT scan and is very concerned about her
25
grandfather's condition. The most appropriate next step is to
A. arrange for an emergent cerebral angiogram
B. call a neurosurgical consult
C. give fresh frozen plasma
D. order an immediate MRI of the head
E. tell the granddaughter that the patient is too old for any treatment and
send him back to the nursing home
Explanation:
The correct answer is B. This is a neurosurgical emergency and a neurosurgical
consult needs to be called. There is no age that is an absolute contraindication
for surgery and it would be wrong to tell the family he is too old for any treatment
and send him back to the nursing home (choice E).
A cerebral angiogram (choice A) would be indicated if you had strong suspicion
for an aneurysm or vascular malformation. This is not the usual cause of a
subdural hematoma.
Since his coagulation studies were already normal, there is no reason to give
fresh frozen plasma (choice C).
A brain MRI (choice D) will not provide any further useful diagnostic information
right now.
A 52-year-old man comes to the emergency department because of "left arm
shaking." He tells you that 2 days earlier he noted left arm paresthesias along the
lateral aspect of his left arm and left 4th/5th fingers while he was reading. He
thinks he may have been leaning on his left arm at the time and the symptoms
resolved after 30 seconds. This morning he noted the same feelings lasting a few
seconds, but then his 4th/5th fingers started shaking rhythmically, which then
migrated to all his fingers, his hand, and then his arm up to his elbow. This
episode lasted a total of 30 seconds. He denies any strange smells or tastes,
visual changes, or weakness. Afterwards, his fingers felt "locked in position" for a
few seconds. Then, he felt as if he did not have control of his hand and had
difficulty donning his socks. He and his wife decided to drive to hospital and in
the car he had trouble putting his seat belt into the socket. Examination and
routine laboratory studies are normal. The most appropriate next step in
management is to
A. discharge the patient to follow up in clinic in 2 weeks
B. obtain a brain MRI
C. obtain an electroencephalogram
D. obtain an orthopedic consult
E. order electromyography and nerve conduction studies
26
Explanation:
The correct answer is B. This history is typical of a simple partial seizure. A focal
brain lesion must be ruled out with an MRI or CT scan.
It would be wrong to discharge the patient to follow up in clinic in 2 weeks
(choice A) without at least a CT scan and preferably an MRI.
Although he probably had a seizure, obtaining an electroencephalogram (choice
C) at this point will not be as helpful as an MRI.
This is unlikely to be a peripheral nerve problem and an orthopedic consult
(choice D) or electromyography and nerve conduction studies (choice E) are not
indicated.
A recently published study examined the efficacy of a new drug, Nomomigrane, for
treating migraine headaches. For this randomized, double-blind study, 200 migraine
sufferers were give Nomomigrane for 6 months, while another 200 migraine sufferers
were given a placebo. At the end of the 6 months, all subjects reported whether or not
they had been migraine free for the last 3 months. A key table from this study is
presented below.
The report included the statement that Nomomigrane worked better than the placebo
and assigned a significance level of p < .001. Based on this and other studies,
Nomomigrane was recently approved by the FDA for the treatment of resistant
migraine headaches. You are considering prescribing Nomomigrane for a patient
suffering from severe migraines. As the treatment option is presented the patient asks,
" I know this worked in the research, but what is the chance that the drug will really
work for me?" Based on the presented research the most appropriate response is:
A. "I'm very excited about the potential for this new drug and think you will be
very pleased with the results."
B. "One can never tell about these things before hand. We'll just have to try it
and see."
C. "The drug was reported effective in 6 out of 10 patients."
D. "The results are strong and convincing. You can have every confidence that
this drug will work for you."
E. "The statistical significance was high in the study. The chance of the drug
27
not working for you is less than 1 in a 1,000."
F. "This treatment will cut the number of migraines you have been
experiencing in half."
G. "Why do you ask? Do you have concerns about my treatment
recommendation?"
Explanation:
The correct answer is C. This question asks about clinical efficacy. In the presented
study, 120 of the 200 patients given the drug did not report migraines in the
measurement period (120/200 = 60%).
Expressing enthusiasm (choice A and D) about a prescribed treatment does help to
engender confidence in the treatment on the part of the patient. However, this
response does not answer the patient's question.
Of course, individual patients may be the exception to the rule (choice B), but this
research tells us that the drug is likely to be effective. The probability is that the drug
will work for this patient.
Statistical significance is high, less than one per 1,000 chance of Type I error.
However, statistical significance tells us little about clinical significance. Knowing the
"p-value" tells us nothing about the number of patients who get better or the chance
that an actual patient will get better (choice E).
The treatment group did report half the migraines compared with the placebo group,
but this is a misuse of this ratio. The study did not measure the reduction in
migraines within each person. Rather, it examined the presence/absence of
migraines between the treatment and control groups. In short, this study does not tell
us what sort of reduction in migraines an individual patient might expect, therefore
(choice F) is incorrect.
Inquiring after a patient's concerns over treatment (choice G) is a reasonable tack,
but does not answer the question posed by the patient. If the patient asks a question,
you should try to answer it.
A 32-year-old woman presents to your outpatient clinic with a several week
history of weakness in her left arm, and a more acute visual disturbance. She
states that her history actually dates back to approximately 6 months ago. She
had a numbness in her right leg that lasted for approximately 3 weeks, but then it
resolved spontaneously. Then approximately 3 weeks ago she began to have
weakness in her left arm when trying to perform some household tasks. Two
days ago she developed acute monocular visual loss in the right eye with
periocular pain. She has no significant past medical history. Her temperature is
37.2 C (99 F), blood pressure is 120/80 mm Hg, pulse is 78/min, and respirations
are 14/min. On ophthalmologic examination, she has an afferent pupillary defect
in the right eye and her visual acuity is 20/20 on the left, and 20/200 on the right.
Her extraocular movements are normal, but cause pain in the right eye. She has
3/5 grip strength in her left hand and the remainder of her motor and sensory
examination is normal. An MRI of her brain and spine shows multiple
hyperintense lesions on T2-weighted images in the white matter tracts as well as
her right optic nerve. This patient will likely benefit from therapy with
28
A. intravenous acyclovir
B. intravenous ceftriaxone after performing a lumbar puncture
C. intravenous methylprednisolone
D. no therapy will be beneficial to this patient
E. radiation therapy to treat the multiple lesions seen on the magnetic
resonance scan
Explanation:
The correct answer is C. This patient is presenting with a fairly classic history for
multiple sclerosis (MS), which is characterized by multiple neurologic symptoms
separated in space and time. That is to say, a single central nervous system
lesion could not explain the signs and symptoms that she has manifested over
the past several months. Her ocular symptoms are very characteristic of optic
neuritis, which is one of the most common manifestations of MS. Furthermore,
she has the classic MRI findings of multiple sclerosis showing multiple plaques
on white matter tracts. One of the mainstays of treatment in MS is corticosteroid
therapy, and it is particularly useful in the acute setting with optic neuritis.
Intravenous acyclovir therapy (choice A) is often used in herpetic encephalitis
that classically affects the temporal lobes of the brain in younger patients and
often presents with seizures.
Intravenous ceftriaxone (choice B) is good initial therapy for acute community
acquired bacterial meningitis. This clinical presentation is not consistent with
meningitis.
No therapy (choice D) is clearly not appropriate for the reasons discussed
previously.
Radiation therapy (choice E) is not used in the treatment of MS, and moreover,
this clinical scenario is not consistent with any form of malignancy.
You are visited in your office by a 40-year-old woman who began experiencing
nausea, vomiting, and numbness in left hand and foot 1 week ago. Today she
began to feel "crescendo pain" in the right retroorbital area. She has had a
headache for a few months that she describes as throbbing and positional,
particularly when she bends forward. The headaches are especially intense in
the morning and at times they have woke her up at night. On examination the
only deficits she has are loss of double simultaneous tactile stimulation and a left
lower facial droop when smiling. The most appropriate next step in management
is to
A. administer intravenous prochlorperazine in your office
B. get her to the emergency department for neurologic evaluation as soon
as possible
C. give her a prescription for zolmitriptan and send her home
D. make an appointment for her to see a neurologist next month
29
E. order an electroencephalogram to rule out seizures
Explanation:
The correct answer is B. The headache is typical of that caused by intracranial
hypertension. Additionally, she has focal neurological symptoms and signs. This
is particularly concerning for a brain tumor or hemorrhage. This should be
evaluated as soon as possible. An appointment next month (choice D) is too
late.
Intravenous prochlorperazine (choice A) is a good treatment for status
migrainous, however this history is atypical for such a diagnosis and more
serious problems should be ruled out first in the emergency department.
Zolmitriptan (choice C) is a treatment for migraines. This history is not typical for
migraine and zolmitriptan is relatively contraindicated in patients with complex
migraine.
This history is very atypical for seizures and an electroencephalogram (choice
E) is not likely to provide useful information in this case.
An 86-year-old man with severe Alzheimer's disease is scheduled to undergo a left carotid
endarterectomy. The patient is on the medical service recovering from a fall sustained during
a syncopal episode. His other past medical history is significant for pharyngeal dysmotility
and reflux disease. He has no known coronary artery disease. In the evaluation of that
episode, it was determined that he had a 99% stenosis of his left internal carotid artery and a
100% occlusion of his right internal and external. This was thought to be the cause of his
syncope. The patient is moderately demented and is fed soft solids and thick liquids with the
help of an aide. In speaking with the anesthesia team, you learn that the patient cannot have
the procedure done under a cervical plexus block due to his dementia and he will therefore
require a general anesthetic. In discussing the risks of the procedure with the patient's
daughter (his health proxy), a serious postoperative complication that this patient is at
especially high risk for is
A. aspiration pneumonitis
B. deep venous thrombosis
C. delirium
D. myocardial infarction
E. need for a tracheostomy
Explanation:
The correct answer is A. For the elderly, the risks of surgery and anesthesia are often
heightened and much of this increased risk stems for comorbid conditions more present in
the elderly. This patient is a high aspiration risk even while awake. After general anesthesia
however, his risk is dramatically elevated such that aspiration of significant quantities of
stomach contents are possible leading to pneumonitis. If the pneumonitis progressed to
pneumonia, a patient such as this, debilitated and moderately demented, would certainly
need a tracheostomy (choice E). The patient's risk however is for the aspiration, not
necessarily the pneumonia or prolonged intubation.
30
All postoperative patients are at risk for deep venous thrombosis (choice B). This patient's
risk is no greater because of his comorbid conditions and his surgical procedure is at the
low risk end of the spectrum since his ambulation should not be limited by the incision in his
neck.
Delirium (choice C) is certainly troublesome for the patient and family, but is not serious.
Elderly patients are more likely to be confused in hospital settings (so-called sundowning)
as well as postoperatively. This patient is at high risk for both given his dementia but again,
these concerns are not serious compared to the possibility of aspiration. The aspiration can
be guarded against, the delirium cannot.
This patient is at low-risk for a perioperative myocardial infarction (choice D) based upon
American Heart Association stratification guidelines that all medical house officers carry in
their jacket pockets. The surgical procedure is classified as low risk. Given the patient's lack
of prior coronary history, he is also classified as low risk. The combination of the two, make
this patient for this procedure a very low-risk candidate for a possible myocardial ischemic
event.
You are working in the emergency department when a 40-year-old woman
presents with a 24-hour history of a severe headache that began as she was
lifting a bucket of water. The pain was abrupt in onset, located in the front of her
head, and has been constant and non-throbbing. She says that now her neck
has become a "bit stiff." She sometimes experiences migraine headaches around
the time of her menstrual period, but says that this is different than the usual
headaches. Her last menstrual period was 2 weeks ago. Her temperature is 37.0
C (98.6 F). She has mild photophobia and discomfort with neck flexion. A CT
scan of the head is normal. A lumbar puncture is performed and the opening
pressure is 22 cm H2O. The cerebrospinal fluid shows 7,000 red blood cells in
tube 1 and 7,200 red blood cells in tube 4. There are 9 white blood cells in each
tube. The fluid is xanthochromic. The next step in managing this case is to
A. arrange for a cerebral angiogram and call a neurosurgical consult
B. give her a prescription for sumatriptan and send her home
C. immediately begin therapy with intravenous acyclovir
D. immediately begin therapy with intravenous ceftriaxone
E. repeat the lumbar puncture
Explanation:
The correct answer is A. This patient probably has a subarachnoid hemorrhage.
She must be evaluated for an aneurysm and a neurosurgical consult must be
obtained.
Sumatriptan (choice B) is a treatment for migraine. The history and
cerebrospinal fluid results do not support a diagnosis of migraine.
Intravenous acyclovir (choice C) would be used to treat herpes encephalitis.
Although there are often red blood cells in the spinal fluid of such patients, the
31
overall history is more consistent with a subarachnoid hemorrhage than herpes
encephalitis.
This does not appear to be bacterial meningitis. It is not emergent that
ceftriaxone (choice D) be given in this case.
Repeating the lumbar puncture (choice E) is not going to help with the diagnosis
or treatment.
A 13-year-old boy is brought to the emergency department because while sitting
at a rock concert with friends he suddenly fell out of his seat to the ground and
appeared to be unconscious. He is now awake and alert and denies any
palpitations or lightheadedness preceding the incident. His past medical history
and family history are non-contributory. His blood pressure is 120/80 mm Hg and
pulse is 65/min, lying down and sitting up. A physical examination and
electrocardiogram are unremarkable. The most appropriate management is to
A. observe in the emergency department and perform serial
measurements of orthostatic blood pressure and heart rate
B. order a complete blood count and blood culture
C. order a toxicology screen
D. perform an echocardiogram
E. send the patient home and advise him to return if it happens again
Explanation:
The correct answer is C. Whenever a patient arrives with a history of a syncopal
episode, especially in a teenager, drug exposure must be considered. In this
case, it is most likely that the syncope was secondary to exposure to a
recreational drug such as cocaine, hallucinogens, or marijuana. Drug exposure
should be considered with young children as well because they can accidentally
ingest a parents' medications if left within reach.
This history is not consistent with that of orthostatic hypotension. If the patient
had a syncopal episode because of orthostatic hypotension one would expect to
hear about a history of lightheadedness, palpitations, prolonged standing,
fainting upon rising, and dehydration. He should be evaluated for the cause of
the episode and therefore, observation in the emergency department and
performing serial measurements of orthostatic blood pressure and heart rate
(choice A) is incorrect. Since this history wasn't obtained it makes the diagnosis
of postural/orthostatic hypotension unlikely.
There is no evidence that this episode of syncope was in any way related to an
infectious cause and therefore a CBC and blood culture (choice B) is
unnecessary.
Whenever a patient has syncope from a sitting or recumbent position it is
worrisome. An arrhythmia must be considered, such as long QT, WolffParkinson-White, AV block, atrial fibrillation/flutter, and ventricular
fibrillation/flutter. An electrocardiogram was done in this case which was
32
negative. This does not rule out an arrhythmia as the primary cause because a
baseline electrocardiogram might be normal. Secondary to the history in this
case, it is more likely that a toxicology screen would be helpful first. If the tox
screen turns out to be negative then a consult with a pediatric cardiologist would
be indicated. But an echocardiogram (choice D) is not necessary at this time.
As apparent by the discussion here, a work up for the syncope is reasonable
and therefore sending the patient home (choice E) is not proper management of
this case.
A 21-year-old woman comes to the office because of 6 months of intermittent
headaches. She reports having headaches in the past, but these are much more
severe. The headaches are described as throbbing, unilateral, with associated
photophobia. They are typically noticed around the time of her menses but are
also exacerbated during stressful times such as her midterm exams. There is not
an aura preceding any of these headaches. She has tried 200 mg of ibuprofen
with minimal relief. Currently she is headache free. Physical examination is
unremarkable. Her neurological examination is non-focal. The most appropriate
pharmacotherapy to treat this patient's headaches is
A. amitriptyline
B. dihydroergotamine
C. indomethacin
D. sumatriptan
E. verapamil
Explanation:
The correct answer is C. NSAIDs are first-line therapy against migraine
headaches. Even if a patient tells you that that they have tried NSAIDs without
relief, you must first inquire if they have taken an appropriate dose prior to
providing other therapy. For example, no headache relief with 200 mg of
ibuprofen is not a treatment failure. First-line therapy for migraines is a trial of
moderate to high-dose NSAIDs along with environmental controls such as
placing the patient in a darkened, quiet room until symptoms resolve. All of the
other agents mentioned below can be effective for migraines but they are
definitely second-line agents or used for migraine prophylaxis.
Amitriptyline (choice A) is a tricyclic antidepressant, which has a role in
prophylaxis of tension and migraine headache but is not useful in acute
treatment.
Dihydroergotamine (choice B) is a potent vasoconstrictor with less peripheral
vasoconstriction than the above. Its advantages are that it comes in IV/IM or
intranasal form. It should also be used with caution in patients with coronary
artery or peripheral vascular disease.
Sumatriptan (choice D) is another potent vasoconstrictor which can be delivered
intranasally or subcutaneously. It should not be used in patients with CAD or
uncontrolled hypertension. It acts by binding to 5-HT1 receptors selectively.
33
Verapamil (choice E) is a calcium channel blocker which can be used as
prophylactic treatment. They have been shown to decrease the frequency of
migraines but they do not affect the severity or duration of attacks.
A 19-year-old woman is admitted to the hospital with severe headache and
photophobia. She reports that over the past few days she has had intermittent
fevers and an "excruciating" headache. This morning, she was unable to lift her
head from the pillow and the lights of the room hurt her eyes. She came to your
office and you admitted her to the hospital for possible viral meningitis. Her
temperature is 39.4 C (102 F), blood pressure is 120/70 mm Hg, pulse is 90/min,
and respirations are 18/min. Physical examination shows photophobia, a positive
Kernig sign, clear lungs, and a normal abdomen. She is started on broadspectrum antibiotics and consents to a lumbar puncture (LP). The most important
consideration prior to beginning the procedure is
A. dural tear
B. increased intracranial pressure
C. pain during neck flexion for the procedure
D. possible bacterial meningitis
E. presence of bacteremia
Explanation:
The correct answer is B. For all patients requiring an LP for any reason, the
paramount concern is the presence of increased intracranial pressure. The
importance of this lies with the theoretical possibility that withdrawing CSF can
precipitate herniation. Therefore, for many patients in many institutions, a head
CT scan without contrast will be obtained to look for signs of increased ICP such
as flattening of the sulci, shrunken ventricles, or a midline shift.
A dural tear (choice A) is common during LP procedures and relates to both the
gauge of the needle and the type of tip. The smaller the needle gauge, the less
likely it is to tear the dura. It is the dural tears that are believed to be the cause
of postdural puncture headaches. In order to minimize this, anesthesiologists
and emergency room physicians will use 22 or 25 gauge needles for LP
procedures.
Pain during neck flexion for the procedure (choice C) is uncomfortable for the
patient, but is of no genuine concern from a medical perspective. This pain
should not prevent an LP from being performed.
Possible bacterial meningitis (choice D), other causes of meningitis, and a
subarachnoid hemorrhage are all indications for an LP. Possible meningitis is
not the most important consideration prior to beginning the procedure, increased
intracranial pressure is.
While the presence of bacteremia (choice E) is often a contraindication to
performing other procedures such as indwelling line placements or epidural
34
catheters, since the CSF is also infected, there is no contraindication to
bacteremia for an LP. In patients where the only source of infection is the blood,
introducing blood into the CSF (for example by performing an epidural) is a risk
that patients should not be exposed to for the pain relief benefits of an epidural
catheter.
A 71-year-old man is brought to the emergency department by his wife who
states that he has seemed "confused" ever since he woke up at 8 a.m. She had
last seen him the evening before when the couple retired to bed and she insists
that he was "his normal self" at that time. He is now unable to provide a history,
but his wife reports that he has never had such an episode in the past. He has a
past medical history of a myocardial infarction 2 years ago, hypertension that is
poorly controlled, and smoking. His wife denies any other medical problems. His
family history reveals that his father had hypertension and a myocardial infarction
at the age of 45 and his brother also suffers from hypertension. The patient is a
retired truck driver who lives with his wife, smokes 1 pack of cigarettes per day,
and drinks 2-3 cans of beer per day. His temperature is 37.0 C (98.6 F), blood
pressure is 192/105 mm Hg, pulse is 96/min, and respirations are 18/min. He is
in no distress. He has normal heart and lung sounds. He is awake, alert, and
attentive to you. He follows all of your commands. He is unable to name simple
objects (for example, he says "clock" when you show him a watch, and says
"dog" when you show him a picture of a cat). He does not repeat phrases well.
For example, he says "no its or buts" when you ask him to repeat "no ifs, ands, or
buts." The most likely diagnosis in this case is
A. Broca's aphasia
B. delirium
C. dysarthria
D. Wernicke aphasia
E. Wernicke encephalopathy
Explanation:
The correct answer is A. Aphasia is a disorder of language that is acquired from
a lesion in the brain. Aphasia must be differentiated from delirium, dysarthria,
and psychosis. Expressive aphasia is caused by a lesion involving Broca's area,
the posterior part of the inferior frontal gyrus. Broca's aphasia, as it is also
known is characterized by intact auditory and written comprehension, but
impairment of fluency, naming, repetition, and writing. Patients frequently
produce paraphasic speech, which is characterized by substitutions of words or
sounds, such as "dog" for "cat" and "band" for "hand", respectively. This type of
aphasia is frequently associated with right hemiparesis or hemisensory loss and
sometimes with apraxia of the left limbs. Aphasia is frequently caused by stroke,
as was the case in this clinical scenario. The patient has many stroke risk
factors (age, gender, hypertension, coronary artery disease, smoking, alcohol
use, family history of early coronary artery disease) and his deficit is new,
suggesting a sudden onset, which is what characterizes stroke.
35
Delirium (choice B) is incorrect. Delirium is an acute confusional state
characterized by fluctuating levels of wakefulness and attention as well as
psychomotor overactivity and hallucinations. Aphasia is not a component of
delirium.
Dysarthria (choice C) is incorrect. Dysarthria is a disorder of speech, the
articulation and phonation of sounds. This is different than aphasia, which is a
disturbance of language.
Wernicke aphasia (choice D) is incorrect. Receptive, or Wernicke aphasia, is
caused by a lesion in the posterior part of the dominant superior temporal gyrus.
It is characterized by fluent speech with impaired naming and repetition. This
speech is sometimes described as "word salad" because it is fluent but devoid
of meaning. Comprehension is impaired, which is the most important distinction
from Broca aphasics, who are able to comprehend.
Wernicke encephalopathy (choice E) is incorrect. Wernicke encephalopathy is a
clinical triad of encephalopathy, ataxia, and ocular motor disturbance that is
caused by thiamine deficiency in alcoholics. Aphasia is not a component of
Wernicke encephalopathy.
A 75-year-old woman with atrial fibrillation comes to the emergency department
because of a 2-hour history of right body weakness and slurred speech. The
onset was sudden while she was walking her dog. She has no complaints of
word finding difficulties, no dysesthesia, and no headaches. She says that she
lives alone, is generally very healthy besides her "heart problem," and takes
multivitamins and warfarin. Her blood pressure is 190/95 mm Hg. Her pulse is
irregularly irregular. Physical examination shows left-sided neglect with slurred
speech and weakness of the right body; face and upper extremity worse than
lower extremity. Routine chemistries and cell counts are normal. Her INR is 1.7.
The most appropriate next step in management is to
A. administer tissue plasminogen activator
B. call a vascular surgery consult for possible endarterectomy
C. order a CT scan of the head
D. send her for a cerebral angiogram
E. start her on heparin
Explanation:
The correct answer is C. This is a good history for cardioembolic stroke —
sudden onset, cortical symptoms, atrial fibrillation, subtherapeutic INR. The
immediate goal should be to rule out an intracranial hemorrhage and confirm the
diagnosis.
Tissue plasminogen activator (choice A) is the treatment for acute stroke in
specific circumstances. We are not sure yet that this is a stroke. It may be an
intracranial hemorrhage which would be a contraindication for tissue
plasminogen activator. Additionally, an elevated INR in a patient on warfarin is a
36
contraindication for tissue plasminogen activator.
Carotid endarterectomy (choice B) is indicated for some cases when a transient
ischemic attack or stroke is believed to be caused by carotid artery narrowing.
We do not know what caused her event and this procedure would rarely be
done emergently.
A cerebral angiogram (choice D) would be indicated if you had strong suspicion
for an aneurysm or vascular malformation. There is no reason to believe one of
these is causing her symptoms.
Heparin (choice E) may be indicated if there is not an intracranial hemorrhage.
This must first be established by CT or MRI.
A 39-year-old man comes to the office because his coworkers have been saying
that his face "is lopsided" for the past 2 days. He says that he does not spend
much time looking in the mirror so he has not really noticed a cosmetic problem,
but he did have pain behind his ear a few days ago and his wife has been
making fun of him for drooling lately. He complains that his left eye has been
drier than usual and he has had to use lubricating drops. He recently returned
from a month long camping and hiking trip through the beautiful wooded regions
of Connecticut. You have treated him for contact dermatitis and the "flu" in the
past, but you have not seen him in a couple of years. The most appropriate way
to test for a facial nerve palsy is to
A. ask him to clench his teeth while you palpate the masseter muscle
B. ask him to show you his teeth
C. have him close his eyes and tell you when you are touching his cheek
with gauze
D. have him turn his head to the right against your hand
E. touch the posterior pharyngeal wall with an applicator stick
Explanation:
The correct answer is B. This patient's symptoms are consistent with a facial
nerve palsy that may be caused by Lyme disease (from his camping trip). To
test the facial nerve, you should ask the patient to show you his teeth. If the left
side of his face is drawn to the right, he most likely has a left-sided facial palsy.
Other facial nerve tests include asking him to wrinkle his forehead, puff out his
cheeks, and hold his eyes shut as tightly as possible as you try to carefully pry
them open.
If this patient had a trigeminal nerve palsy, you should ask him to clench his
teeth while you palpate the masseter muscle (choice A). During this maneuver,
his jaw would deviate to the same side as the lesion. Also, he would complain of
sensory abnormalities on his forehead, cheek, and jaw and paroxysms of pain
on his chin, lips, cheeks, and gums.
Sensory abnormalities can be elicited by having him close his eyes and tell you
37
when you are touching his cheek with gauze (choice C). He does not have
these sensory symptoms.
If he had a left-sided spinal accessory nerve palsy, you should have him turn his
head to the right against your hand (choice D). The dysfunction would manifest
with the patient being unable to do this, or weakness on this side as opposed to
the other side. He does not complain of weakness in the head, neck, or
shoulder.
The gag reflex of the glossopharyngeal nerve is tested by touch the posterior
pharyngeal wall with an applicator stick (choice E). Symptoms of
glossopharyngeal nerve lesions are sensory dysfunction of the pharynx, loss of
taste on the posterior third of the tongue, and a partially dry mouth.
A 37-year-old man is hospitalized in an intensive care unit after suffering major
head trauma in a motor vehicle accident. The patient was struck by a car two
days ago and was found on the scene to be non-responsive with a Glasgow
Coma Score of 4. He was intubated at the scene and transferred to the hospital.
In the prior two days, his mental status is unchanged and he remains intubated
and ventilated. You have documented a discussion you had with the patient three
months ago during an admission for pneumonia. During that discussion the
patient clearly stated that he would want to be maintained on life support only if
he were likely to regain a meaningful quality of life. You and the medical team
believe that he does not have a significant chance of regaining an acceptable
level of function. The brother and sister maintain that the situation is reversible
while the wife continues to desire the withdrawal of care in fulfillment of her
husbands' wishes. The most appropriate next step is to
A. arrange a family meeting hoping to resolve the patient's previously
expressed wishes with those of the children and husband
B. begin the withdrawal of care despite the reservation of the siblings
after discussion with the hospital lawyer
C. consult a psychiatrist to speak to the siblings
D. notify the department of social services for the question of spousal
abuse
E. refer the case to the ethics committee for review
Explanation:
The correct answer is A. Although it is clear that you are ethically bound to
follow the patient's wishes, which in this care appears to be the withdrawal of
care, it is always better to do so with the resolve of the family. Often times,
feelings such as guilt will drive family members to insist on seemingly
unreasonable or inappropriate action. A thoughtful discussion, whereby the
family members are allowed to express their reservations in a supportive setting,
often will produce a resolution among previous disparate views.
While withdrawing care (choice B) appears to be what ultimately is the most
appropriate action, it is worth trying first to have all of the interested parties “on
the same page” before preceding if this resolution can be achieved in a timely
38
manner without causing suffering on the patient's part.
Whereas consult services such as psychiatry (choice C) may be helpful on
selected occasions, the first attempts to resolve the conflict should fall on the
primary medical team and primary care physicians.
While physicians have the positive duty to report suspected abuse, there is no
indication from the information present that the wife has abused her husband
(choice D).
Ethics committees (choice E) may be helpful in resolving conflict, but, again, the
primary medical providers should first attempt to resolve conflict in order to
respect the patient's wishes in a thoughtful and timely manner.
A 42-year-old woman is brought to the emergency department after being hit on
the side of the head with a pipe. She was brought in by a witness who says that
the patient was unconscious for about 2 minutes. On examination, she has a
normal mental status, but does not remember being hit. Neurologic examination
is otherwise completely normal. There is a minor superficial left temporal
laceration. A complete blood count, electrolytes, and coagulation studies are
normal. A CT scan of the head shows a very small left epidural hematoma. After
caring for her superficial wound, the next most appropriate action is to
A. admit her to the hospital for observation
B. arrange for a cerebral angiogram
C. discharge her to home from the emergency department
D. get an MRI of the brain
E. perform a lumbar puncture
Explanation:
The correct answer is A. Patients with new intracranial bleeds, especially
epidural hematomas, must be watched closely for 24-48 hours. Epidural bleeds
are notorious for worsening quickly following an apparent recovery from the
original trauma.
If she had subarachnoid blood, which was not obviously caused by trauma, you
would arrange for a cerebral angiogram (choice B) to look for an aneurysm or
vascular malformation.
This patient has an epidural hematoma, which can quickly worsen and therefore
she needs to be admitted for observation. It would be completely inappropriate
to discharge her home from the emergency department (choice C).
A brain MRI (choice D) would not give you any further useful information
because we already know from the CT scan that she has an epidural
hematoma.
A lumbar puncture (choice E) is used to look for subarachnoid blood not seen
on CT, but still suspected clinically. This patient has evidence of an epidural
39
hematoma and there is no reason to suspect that she also has a subarachnoid
hemorrhage.
A 67-year-old physician with diabetes mellitus and hypertension comes to the
emergency department following a "transient episode of alexia." She got up in
morning and felt fine but as she was showering she felt slightly dizzy and
lightheaded and "just did not feel like her usual self." As she was giving herself
insulin, she had difficulty with the numbers on the syringe. She could see them
but she found that she had to count each mark on the syringe to be sure she was
giving herself 10 units. She checked her blood sugar, which was 110 mg/dL. She
went to have breakfast and as she tried to read the paper she found that she
could see the letters but was unable to make sense of the letters. She spoke to
her husband, who thought she looked puzzled. He did not note any facial
asymmetry and had no trouble understanding what she was saying. She was
also able to understand him. She tried to take her blood pressure and noticed
that when she was manipulating the bulb with her right hand, she was unable to
feel the screw. She does not think her hand was weak or clumsy, but rather that
she could not feel things. She denies any visual changes and when she looked at
the clock she could see all the numbers but she couldn't tell what time it was. The
symptoms resolved within 30 minutes of when she first realized she was unable
to read the paper. Now she has a mild right-sided headache. Physical and
neurological examinations are normal. The next step in managing this patient is
to
A. admit her to the hospital for observation and cerebrovascular
evaluation
B. call a vascular surgery consult for possible endarterectomy
C. give her a prescription for sumatriptan and send her home
D. start warfarin and send her home
E. send her home and tell her that she has fully recovered
Explanation:
The correct answer is A. The history is good for a transient ischemic attack. She
should have a full cerebrovascular evaluation. This potentially could be done as
an outpatient, however this was not a choice and it is preferable to do as an
inpatient.
Carotid endarterectomy (choice B) is indicated for some cases when a transient
ischemic attack or stroke is believed to be caused by carotid artery narrowing.
We do not know what caused her event.
Sumatriptan (choice C) is a treatment for migraine. We do not know if this was a
migraine and she has 2 significant risk factors for cerebrovascular disease.
Additionally, sumatriptan is relatively contraindicated in patients with complex
migraine.
Warfarin (choice D) may be used for secondary prevention of stroke following a
transient ischemic attack in certain cases, particularly if a cardiac embolus is to
40
blame. However, we do not yet know what caused her event.
Although she appears to have fully recovered from the episode, this may have
been a transient ischemic attack, putting her at higher risk for a stroke. She
should have a full cerebrovascular evaluation and possibly begin secondary
prevention. You should not send her home and tell her that she has fully
recovered (choice E).
A 31-year-old woman comes to your office complaining of weight loss, anxiety,
hot flashes, and diarrhea. She also reports that she is unable to fall asleep at
night and wakes early in the morning. She works as a nurse at the local
community hospital. Her temperature is 37.0 C (98.6 F), blood pressure is 110/70
mm Hg, pulse is 110/min, and respirations are 15/min. She appears anxious and
jittery throughout the interview. Physical examination shows warm, moist skin, a
diffusely enlarged thyroid gland, and a slight decrease in her peripheral vision
bilaterally. She does not have lid lag or a "stare". Her reflexes are hyperactive but
symmetric. Suspecting hyperthyroidism you order the following labs: Thyroid
stimulating hormone (TSH) 9 mU/L, thyroxine (T4) 16 nmol/L, triiodothyronine
(T3) 204 nmol/L. The most appropriate next step is to
A. continue further history taking to exclude factious hyperthyroidism
B. order antithyroglobulin and antithyroid peroxidase antibodies
C. order thyroid stimulating hormone receptor antibody assay
D. schedule an MRI of the pituitary gland
E. schedule radioiodine thyroid scintiscanning
Explanation:
The correct answer is D. MRI of the pituitary gland would likely reveal an
adenoma, which is likely secreting TSH. This patient is obviously hyperthyroid
by history. She reports anxiety, weight loss, difficulty sleeping, hot flashes, and
diarrhea. Her exam reveals a nervous woman with a goiter, moist skin,
tachycardia, and hyperreflexia. She lacks the exophthalmos seen in Graves
disease. The "mild decrease in peripheral vision" suggest a pituitary mass. Her
thyroid function tests reveal inappropriate secretion of TSH by the pituitary
despite an elevated T3 and T4. This suggests that the pituitary gland is
functioning autonomously without the proper feedback from T3 and T4.
Normally, high levels of thyroid hormone would be expected to suppress TSH. It
is important to recognize that the TSH in this patient could have been within
normal range, but in the presence of elevated thyroid hormone this would be
inappropriate as well. An MRI of the pituitary gland would probably reveal a
macroadenoma.
Factious disorders (choice A) are an important cause of illness and the USMLE
likes to ask questions about them. A case with multiple laboratory
inconsistencies, especially if the patient is a health care worker, might make you
suspect a factious disorder. In this case, ingestion of levothyroxine would lead to
elevated thyroid hormone levels but would suppress TSH. Remember that this is
41
a diagnosis of exclusion.
Antithyroglobulin and antithyroid peroxidase (choice B) antibodies are helpful in
confirming a diagnosis of Hashimoto thyroiditis (hypothyroidism). They can also
be elevated in Graves disease. These antibodies are elevated in a small
percentage of euthyroid patients as well.
TSH receptor antibodies (choice C) are positive in approximately 80% of
patients with Graves disease. They are a reasonable test to order if you suspect
Graves disease but a negative test certainly wouldn't exclude Graves as your
diagnosis.
Radioactive uptake scans (choice E) can be part of the workup for Graves
disease because it not only helps confirm the diagnosis by showing a diffuse
increase in uptake, but later it is useful to determine the dose of radioactive
iodine that would be necessary if the patient were to opt for radioactive ablation
as their form of therapy. In this patient, one would likely see a diffuse uptake of
radioactive iodine because the gland is actively producing excess thyroid
hormone. It would not help in ruling out a pituitary etiology.
A 32-year-old HIV-positive intravenous drug abuser is admitted to the hospital
after being found on the city sidewalk having a generalized seizure. A CT scan of
the head is performed and shows several rim-enhancing lesions with minimal
mass effect. You notice in his chart that he takes multiple "HIV medications" but
he is unsure of the exact names and his grandmother died of a ruptured cerebral
aneurysm. The most appropriate next step in management is to
A. get a cerebral angiogram
B. order a ventricular cerebrospinal fluid (CSF) aspiration
C. perform a lumbar puncture and include cerebrospinal fluid for Epstein
Barr virus (EBV) PCR in tests ordered
D. stop all antiretroviral therapy
E. treat with intravenous acyclovir
Explanation:
The correct answer is C. The most common etiologies of rim-enhancing brain
lesions in AIDS patients are primary CNS lymphoma (PCNSL) and Toxoplasma
gondii infection. Other etiologies such as bacterial or fungal abscess are also
possible. CSF EBV PCR test is highly sensitive and specific for PCNSL.
A cerebral angiogram (choice A) should be done if you suspect an aneurysm or
vascular malformation. These are unlikely in this case.
Since there is no mass effect it is safe to do a lumbar puncture. Therefore, a
ventricular cerebrospinal fluid (CSF) aspiration (choice B) is not necessary.
There is no reason to stop all antiretroviral therapy (choice D).
Intravenous acyclovir (choice E) is used to treat herpes encephalitis, which is
42
unlikely in this case.
A 69-year-old man with diabetes mellitus comes to the emergency department
because of a 12-hour history of a left facial droop, slurred speech, difficulty
swallowing, and decreased balance. He woke up the day earlier feeling
lightheaded. He had no palpitations, diaphoresis, or visual changes. He went
back to bed and at noon awoke with difficulty swallowing and a tingling sensation
on the left side of his tongue. He noticed that he had difficulty with balance that
made walking very difficult. He had no vertigo and ate without difficulty. He
denies word finding difficulties, dysesthesia, or any headaches. His blood
pressure is 180/75 mm Hg and pulse is 60/min. Examination shows slurred
speech and weakness and decreased sensation of the left lower face. His
glucose finger stick is 37 mg/dL. The most appropriate next step is to
A. administer tissue plasminogen activator
B. give him a glass of orange juice
C. give intravenous dextrose immediately
D. order a brain MRI
E. start him on prednisone and acyclovir
Explanation:
The correct answer is C. One of the most common causes of neurological deficit
in elderly patients is a toxic or metabolic disturbance. Hypoglycemia is
particularly common in patients who are being treated for diabetes. These
problems are often easily correctable if caught early. While this patient may
have had a stroke, the first step is to correct his hypoglycemia. Orange juice
(choice B) may also work, but it will not work as quickly and is not the best
choice because he is having difficulty swallowing and so he is at risk for
aspiration.
Tissue plasminogen activator (choice A) is the treatment for acute stroke in
specific circumstances. We do not know that he had a stroke and since his
symptoms are relatively mild and over 3-hours old, tissue plasminogen activator
is not an appropriate treatment.
Stroke is a possibility in this case. Brain MRI (choice D) would be the best test
for this, however it can take a while to get this done and you should first correct
the toxic/metabolic disturbances.
Prednisone and acyclovir (choice E) are the treatment for Bell palsy. His facial
weakness is not the pattern of Bell palsy and Bell palsy does not cause sensory
impairments.
A 29-year-old man comes to the emergency department because of a severe "excruciating
headache" for the past 12 hours. He is an associate professor at a local college and recently
had a 3-day "end of semester party" for some of his favorite students at his "weekend
house." He vaguely recalls that a couple of these students complained of a headache on the
43
last day of their stay, but the previous night was "quite a party." He is generally healthy,
takes no medications, and "drinks a few beers" each night. His temperature is 37.8 C (100.0
F), blood pressure is 150/90 mm Hg, pulse is 70/min, and respirations are 16/min. He has
mild nuchal rigidity and flank tenderness. A careful cardiac examination reveals a midsystolic click. Ophthalmologic examination shows a flat optic disc. During the examination,
he turns to the side and vomits on your shoes. You decide to perform a lumbar puncture and
place him in the lateral decubitus position with his thighs flexed. You introduce the spinal
needle and note that the opening pressure is 230 mm Hg. As you collect the four tubes of
spinal fluid you realize that the fluid is red. You run the tubes to the laboratory and put them
in the centrifuge and one hour later see that the supernatant of the centrifuged cerebrospinal
fluid is a yellowish color. The most likely explanation is
A. acute bacterial meningitis
B. acute viral meningitis
C. a brain tumor
D. a subarachnoid hemorrhage
E. a traumatic lumbar puncture
Explanation:
The correct answer is D. This patient most likely has a subarachnoid hemorrhage, presenting
with a severe headache, and therefore has gross blood in the cerebrospinal fluid and
xanthochromia (the yellow color) of the supernatant of the centrifuged fluid. The
xanthochromia is the result of lysis of red cells and release of their intracellular contents into
the cerebrospinal fluid, which usually occurs about 2-4 hours after a bleed. The elevated
opening pressure can be associated with meningitis, a brain tumor, or a subarachnoid
hemorrhage. It is possible that this patient has polycystic kidney disease because he has
hypertension, mitral valve prolapse, a subarachnoid hemorrhage, and flank pain. An
ultrasound or a CT scan is indicated, and may reveal renal cysts. A CT scan of the head will
most likely show a subarachnoid hemorrhage.
Acute bacterial meningitis (choice A) and acute viral meningitis (choice B) are not usually
associated with blood in the cerebrospinal fluid. It is possible to have xanthochromatic CSF
when the CSF protein is elevated to 150 to 200 mg/dL, however, this patient has many other
signs that indicate that he may have polycystic kidney disease and a subarachnoid
hemorrhage. The headaches that the students complained of were most likely due to heavy
alcohol consumption the previous night, not meningitis.
It is unlikely that this patient has a brain tumor (choice C). While a brain tumor may present
with a headache and it is possible to have xanthochromatic CSF, the acute onset of the
symptoms is more consistent with a subarachnoid hemorrhage. Also, the hypertension, flank
pain, and mid-systolic click are suggestive of polycystic kidney disease, which is associated
with berry aneurysms and subarachnoid hemorrhages.
A traumatic lumbar puncture (choice E) is when a meningeal vessel is penetrated during a
procedure, and usually shows pinkish-red CSF that clears progressively from tubes one to
four. A clear, not yellow, supernatant is generally found after the CSF is centrifuged. This is
most likely because the red blood cells have not had a chance to lyse and release their
intracellular contents into the cerebrospinal fluid
44
A 12-year-old boy is brought to the office because of "really strange episodes"
that have been occurring over the past few months. He tells you that he is unable
to remember the entire event, but he can piece it together from the stories other
people tell him. The episodes start with an internal feeling of detachment, then he
sits motionless and stares at the wall, and finally he begins to make repeated
"picking" movements with both hands. He is very confused for up to a half-hour
following the event. People try to talk to him and wave their hands in front of his
face while this is happening, but he does not respond. The most likely diagnosis
is
A. absence seizures
B. complex partial seizures
C. generalized tonic-clonic seizures
D. myoclonic seizures
E. simple partial seizures
Explanation:
The correct answer is B. This patient has complex partial seizures, which are
characterized by automatisms ("picking" movements with hands, chewing,
swallowing), that are preceded by an aura. The sequence of events is as
follows: aura, behavioral arrest or a motionless stare, automatisms, and postictal
confusion. The patient has an impairment of consciousness. These seizures
occur in a focal region of the brain, often the temporal lobe.
Absence seizures (choice A), also called petit mal seizures, are generalized
seizures that consist of a brief loss of consciousness and automatisms. Postural
control is a maintained. There is no postictal confusion.
Generalized tonic-clonic seizures (choice C) are characterized by a sudden
contraction of the muscles followed by periods of contraction and relaxation, and
finally, relaxation. There is postictal unresponsiveness, excess salivation, and
bowel and bladder incontinence.
Myoclonic seizures (choice D) consist of the sudden contraction of one part of
the body or may involve the entire body. It is caused by cortical dysfunction.
Simple partial seizures (choice E) are characterized by motor, autonomic,
sensory, or psychic symptoms. An aura may precede the seizure.
Consciousness is maintained.
A 27-year-old woman is admitted to the hospital because of optic neuritis of the
left eye that was diagnosed 2 days earlier. She is currently being treated with
intravenous methylprednisolone. The patient denies having had recent
vaccinations or severe cold-like illnesses. A full work up, including an MRI, shows
a lesion of the left optic nerve. Her spinal tap on admission revealed clear
cerebrospinal fluid with a normal protein, glucose of 58, 7 lymphocytes, no
oligoclonal bands, but high IgG of 7.4 (normal below 5.5). Her temperature is
37.0 C (98.6 F). She is oriented and attentive with good memory and judgment.
45
There is no aphasia or dysarthria and there is no right left disorientation. Her
affect is appropriate. Visual acuity is 20/20 bilaterally now. However, she has a
left afferent pupillary defect and left eye color desaturation. There is no
nystagmus, extraocular movement abnormalities, or visual field deficits. Facial
strength and sensation are intact and her hearing is normal. Tongue and palate
are midline. She shrugs both shoulders. Strength is 5/5 throughout. Reflexes are
2+ throughout with downgoing toes. Sensory examination to light touch, pink
prick vibration, position, and temperature are intact. The patient is able to walk on
heels, toes, and in tandem. There is no Romberg sign. The most appropriate
next step in management is to
A. begin therapy with glatiramer
B. begin therapy with interferon beta-1B
C. begin therapy with interferon beta-1A
D. get a neurology consult
E. start her on daily aspirin therapy
Explanation:
The correct answer is D. This patient with optic neuritis needs a more thorough
evaluation, including special CSF studies and a spine MRI. Optic neuritis may
progress to multiple sclerosis (MS), but does not always. This patient should be
further evaluated and followed by a neurologist.
Glatiramer (choice A), interferon beta-1B (choice B), and interferon beta-1A
(choice C) are all treatments for MS. They are primarily used for relapsingremitting MS. However, their use is still evolving and they are best prescribed by
a specialist.
Daily aspirin (choice E) is a treatment for cerebrovascular disease which she
probably does not have.
You are seeing a 28-year-old previously healthy male bodybuilder in the
emergency department for headache evaluation. He tells you that he was lifting
weights yesterday afternoon and then when he got home he developed a nonthrobbing bifrontal headache during dinner. He took some aspirin and lay down
and felt better in about 20 minutes. Then he got up and within 10 minutes of
resuming normal activity the headache returned. He went to bed for the night and
the headache resolved. Today the pain returned, but he feels better now that he
is lying down on a stretcher. Neurologic examination is unremarkable. The most
appropriate initial management of this patient's condition is
A. bed rest and hydration for 1-2 weeks
B. brain MRI
C. intravenous prochlorperazine
D. observation in the hospital
46
E. psychiatry consult
Explanation:
The correct answer is A. This is the presentation for a headache caused by
intracranial hypotension. It is often caused by a dural tear during physical
exertion. The initial management is conservative and includes bed rest and
hydration for 1-2 weeks.
It is not necessary to get a brain MRI (choice B).
Intravenous prochlorperazine (choice C) is a treatment for acute migraine
headache.
Observation in the hospital (choice D) is not necessary. This can be managed
conservatively as an outpatient with rest and fluids.
There is no indication for a psychiatry consult (choice E) as it seems like this
patient has a headache consistent with intracranial hypotension, not a
psychiatric disorder.
S3 NEURO
S3 NEURO
47